SlideShare a Scribd company logo
1 of 64
Contemporary Management of HIV: How
Common Comorbidities Affect ART
Management
This program is supported by an independent educational grant from
ViiV Healthcare.
 Please feel free to use, update, and share some or all
of these slides in your noncommercial presentations
to colleagues or patients
 When using our slides, please retain the source
attribution:
 These slides may not be published, posted online, or
used in commercial presentations without permission.
Please contact permissions@clinicaloptions.com for
details
Slide credit: clinicaloptions.com
About These Slides
Program Director and Core Faculty
Program Chair
Joseph J. Eron, Jr., MD
Professor of Medicine and
Epidemiology
University of North Carolina
School of Medicine
Director, AIDS Clinical Trials
Unit
University of North Carolina
Chapel Hill, North Carolina
David A. Wohl, MD
Professor of Medicine
School of Medicine
Site Leader, AIDS Clinical
Trials Unit-Chapel Hill
University of North Carolina
at Chapel Hill
Director, North Carolina
AIDS Training and Education
Center
Chapel Hill, North Carolina
Co-Director, HIV Services
North Carolina Department
of Correction
Raleigh, North Carolina
Faculty Disclosure Information
Joseph J. Eron, Jr., MD, has disclosed that he has
received consulting fees from Gilead Sciences,
Janssen, Merck, and ViiV and funds for research
support from AbbVie, Gilead Sciences, Janssen, and
ViiV.
David A. Wohl, MD, has disclosed that he has received
consulting fees from Bristol-Myers Squibb, Gilead
Sciences, Janssen, and ViiV and funds for research
support from Gilead Sciences.
Peer Review Disclosure
Barry S. Zingman, MD
Medical Director, AIDS Center
Clinical Director, Infectious Diseases, Moses Division
Professor of Clinical Medicine, Albert Einstein College of
Medicine
Montefiore Medical Center
The University Hospital for Albert Einstein College of
Medicine
Barry S. Zingman, MD, has no real or apparent
conflicts of interest to report.
Program Overview
 Comanaging CVD risk and ART selection in middle-
aged HIV-infected pts
 Managing low bone mineral density and fracture risk
in older pts on long-term suppressive ART
 Managing concomitant ART and HCV therapy in
HIV/HCV-coinfected pts
Case 1: Patient With CVD Risk
Case 1: Pt With CVD Risk
 MJ is a 56-yr-old black woman diagnosed with HIV
infection 6 yrs ago
 She fell out of care 1 yr ago and stopped ART
 1 mo ago, she returned to care and was started on
enalapril for hypertension
 Smokes 1 pack cigarettes per day for > 30 yrs
Slide credit: clinicaloptions.com
Case 1: HIV/ART History
 Laboratory values at the time of HIV diagnosis
– CD4+ cell count 486 cells/mm3
– HIV-1 RNA 12,000 copies/mL
 Started on boosted PI + FTC/TDF
– HIV-1 RNA was suppressed except for occasional blips
 Prior to falling out of care, her last CD4+ cell count was
744 cells/mm3
and HIV-1 RNA was 410 copies/mL
 Stopped ART when her father passed away and she took
on care of her 80-yr-old mother; her mother has since
moved to a nursing home allowing her to address her own
healthcare needs
Slide credit: clinicaloptions.com
 Current presentation today
– BP 142/88 mm Hg
– BMI 27
– Urinalysis: trace protein
– Toxicology screen: negative
– HLA-B*5701 negative
– TC 207 mg/dL, HDL
42 mg/dL, LDL 130 mg/dL,
TG 176 mg/dL
Case 1: Recent History and Current
Presentation
 Returned to care 1 mo ago
– HIV-1 RNA
21,000 copies/mL
– CD4+ cell count
322 cells/mm3
– HCV negative; HBV immune
– Serum creatinine
1.13 mg/dL (MDRD
> 60 mL/min/1.73 m2
)
– HIV genotype: wild-type
virus
Slide credit: clinicaloptions.com
Case 1: Issues for Discussion
 Should MJ be prescribed a statin?
 What ART should MJ be prescribed?
 What else can be done to reduce MJ’s risk for new or
worsening comorbidities?
Slide credit: clinicaloptions.com
 56-yr-old black woman diagnosed with HIV and initiated ART 6 yrs ago
 Fell out of care and discontinued ART 1 yr ago, now returned to care
 HIV-1 RNA 21,000 copies/mL; CD4+ cell count 322 cells/mm3
 HIV GT WT, but was viremic on boosted PI + FTC/TDF 1 yr ago
 HBV immune, HCV negative, HLA-B*5701 negative
 Serum creatinine 1.13 mg/dL (MDRD > 60 mL/min/1.73 m2
); dyslipidemia
 Receiving enalapril for HTN
 Smokes 1 pack cigarettes per day for > 30 yrs
Declines in MI, Stroke in United States:
Kaiser Permanente CA Cohort
After adjusting for stroke risk factors:
No increased risk in HIV+ with
recent CD4+ cell count ≥ 500
cells/mm3
or recent HIV-1 RNA
< 500 c/mL
Increased risk in HIV+ with recent
CD4+ cell count < 500 cells/mm3
or
recent HIV-1 RNA ≥ 500 c/mL
Recent reduced MI rates for HIV+
likely due to:
CVD risk reduction, lipid-friendly
ART, reduced immunodeficiency
1. Klein DB, et al. CROI 2014. Abstract 737. 2. Klein DB, et al.
Clin Infect Dis. 2015;60:1278-1280. 3. Marcus JL, et al. CROI 2014.
Abstract 741. 4. Marcus JL, et al. AIDS. 2014;28:1911-1919.
200
150
100
50
1996-1999
2000-2003
2004-2007
2008-2009
2010-2011
250
0
Stroke Rates by HIV Status and Yr[3,4]
Cases/100,000PYs
400
MIs/100,000PYs
300
200
100
0
1996-99
2000-03
2004-07
2008-09
2010-11
HIV+
HIV-
MI Rates Over Time by HIV Status[1,2]
HIV+
HIV-
Slide credit: clinicaloptions.com
Guesses?
http://tools.acc.org/ASCVD-Risk-Estimator/
Case 1: ASCVD Risk Estimator
Any
Slide credit: clinicaloptions.com
http://tools.acc.org/ASCVD-Risk-Estimator/
Case 1: ASCVD Risk Estimator
Slide credit: clinicaloptions.com
http://tools.acc.org/ASCVD-Risk-Estimator/
ASCVD Risk Estimator:
Recommendations
Adults 40-75 yrs of age with LDL 70-189 mg/dL with no diabetes and estimated 10-yr
ASCVD risk ≥ 7.5% should be treated with moderate- to high-intensity statin therapy (I A)
Slide credit: clinicaloptions.com
 CVD risk scores calculated with data from 2006-2009 for pts in Partners HealthCare
System Cohort[1]
 Analysis of HOPS cohort (n = 2283)found FRS accurately estimated CVD risk but
AHA/ACC and D:A:D underestimated risk[2]
1. Regan S, et al. CROI 2015. Abstract 751.
2. Thompson-Paul AM, et al. Clin Infect Dis. 2016;63:1508-1516.
5-Yr Predicted Rate (%)
Framingham Risk Score[1]
5-YrEventRate(%)
5-YrEventRate(%)
ACC/AHACVD Risk Calculator[1]
5-Yr Predicted Rate (%)
Observed
Predicted
Observed
Predicted
CVD Outcomes Underestimated in HIV-
Positive Pts by Risk Calculators
n = 2270 n = 2152
25
20
15
10
5
0
< 2.5 2.5-4.9 5.0-7.4 7.5-9.9
25
20
15
10
5
0
< 2.5 2.5-4.9 5.0-7.4 7.5-9.9
Slide credit: clinicaloptions.com
Case 1: Issues for Discussion
 Should MJ be prescribed a statin?
 What ART should MJ be prescribed?
 What else can be done to reduce MJ’s risk for new or
worsening comorbidities?
Slide credit: clinicaloptions.com
 56-yr-old black woman diagnosed with HIV and initiated ART 6 yrs ago
 Fell out of care and discontinued ART 1 yr ago, now returned to care
 HIV-1 RNA 21,000 copies/mL; CD4+ cell count 322 cells/mm3
 HIV GT WT, but was viremic on boosted PI + FTC/TDF 1 yr ago
 HBV immune, HCV negative, HLA-B*5701 negative
 Serum creatinine 1.13 mg/dL (MDRD > 60 mL/min/1.73 m2
); dyslipidemia
 Receiving enalapril for HTN
 Smokes 1 pack cigarettes per day for > 30 yrs
 ASCVD 10-yr risk: 18%
ART and Effects on Lipids
TDF ABCTAF
RAL
DTG
ATV/RTV or ATV/COBI
DRV/RTV or DRV/COBI
EVG/COBI
EFVRPV
ETV
Slide credit: clinicaloptions.com
Slide credit: clinicaloptions.com
Gatell JM et al. IAS 2017. Abstract TUAB0102.
ClinicalTrials.gov. NCT02098837.
NEAT 022: Switch From Boosted PI to
DTG in Suppressed Pts With High CV Risk
 International, randomized, open-label phase IV study
– Primary endpoints at Wk 48: proportion with HIV RNA
< 50 copies/mL (ITT), change in total plasma cholesterol
 Baseline NRTI backbones: FTC/TDF, 64.8%; ABC/3TC, 31.3%
Pts with stable HIV-1 RNA
< 50 c/mL on PI/RTV + 2 NRTIs,
high CV risk,* no resistance
mutations, no VF
(N = 415)
Immediate switch to DTG + 2 NRTIs†
(n = 205)
Continue
PI/RTV + 2 NRTIs†
(n = 210)
Deferred switch to
DTG + 2 NRTIs†
Wk 48 Wk 96
*> 50 yrs of age and/or Framingham risk score > 10% at 10 yrs.
†
NRTIs to remain the same throughout study.
 Switching to DTG noninferior to
continuing boosted PI through
Wk 48
Slide credit: clinicaloptions.comGatell JM, et al. IAS 2017. Abstract TUAB0102.
NEAT 022: Key Findings
 Switching to DTG assoc. with
improved lipid profile vs cont.
boosted PI through Wk 48
 No emergent resistance in pts with VF
 No significant differences in grade 3/4 AEs, serious AEs, AE-related d/c
Virologic
Success*
Virologic
Nonresponse
No
Virologic
Data
ITTPopulation(%)
Treatment difference: -2.1%
(95% CI: -6.6% to 2.4%)
4.94.4
100
80
60
40
20
0
93 95
2 0.5
DTG + 2 NRTIs
PI/RTV + 2 NRTIs
DTG + 2 NRTIs
PI/RTV + 2 NRTIs
0.7
-8.7
-11.3
0.5
4.2
2.0 1.1
2.5
0.4
-18.4
-7.7 -7.0
TC Non–
HDL
-C
TG LDL-
C
HDL-
C
TC/
HDL
Ratio
P < .001
P < .001
P < .001
P < .001 P < .001
P = .286
MeanChange
FromBLtoWk48(%)
*HIV-1 RNA < 50 copies/mL.
5
-5
-15
10
0
-10
-20
-25
Case 1: Issues for Discussion
 Should MJ be prescribed a statin?
 What ART should MJ be prescribed?
– What if MJ’s serum creatinine were 1.9 mg/dL and
MDRD were 42 mL/min/1.73 m2
?
 What else can be done to reduce MJ’s risk for new or
worsening comorbidities?
Slide credit: clinicaloptions.com
 HIV-1 RNA 21,000 copies/mL; CD4+ cell count 322 cells/mm3
 HIV GT WT, but was viremic on boosted PI + FTC/TDF 1 yr ago
 HBV immune, HCV negative, HLA-B*5701 negative
 Serum creatinine 1.9 mg/dL (MDRD 42 mL/min/1.73 m2
); dyslipidemia
 Smokes 1 pack cigarettes per day for > 30 yrs
 ASCVD 10-yr risk: 18%
Case 1 Question: Would you prescribe an
ABC-based regimen?
A. Yes
B. No
 HIV-1 RNA 21,000 copies/mL; CD4+ cell count 322 cells/mm3
 HIV GT WT, but was viremic on boosted PI + FTC/TDF 1 yr ago
 HBV immune, HCV negative, HLA-B*5701 negative
 Serum creatinine 1.9 mg/dL (MDRD 42 mL/min/1.73 m2
); dyslipidemia
 Smokes 1 pack of cigarettes per day for > 30 yrs
 ASCVD 10-yr risk: 18%
Potential Association Between ABC Use
and CVD Event Risk: Conflicting Data
 Multiple studies have identified increased risk of MI or
overall CVD events with ABC use, including large
cohort studies, RCTs, and case-control studies[1-10]
– Range of risk estimates across studies: 1.3- to 4.3-fold
increase
 However, several studies have also demonstrated a
lack of increased CVD risk with ABC use, including
large cohort studies, a meta-analysis of RCTs, and a
case-control study[10-14]
 DHHS: consider avoiding ABC in pts at elevated risk
for CVD[15]
References in slidenotes. Slide credit: clinicaloptions.com
TAF in Pts With Renal Insufficiency
 Open-label trial of 242 virologically suppressed pts with stable
eGFRCG 30-69 mL/min switched from TDF and non-TDF
regimens to E/C/F/TAF[1]
– eGFR remained stable through Wk 144 analysis[2]
1. Post FA, et al. CROI 2016. Abstract 680.
2. Podzamczer D, et al. IAS 2017. Abstract MOPEB0288.
Slide credit: clinicaloptions.com
MedianeGFR
(mL/min/1.73m2
)
Wks
eGFR (CKD-EPI, Cystatin C)100
80
60
40
0
Baseline
eGFR, mL/min
> 60 (n = 163)
51-60 (n = 39)
41-50 (n = 20)
31-40 (n = 14)
≤ 30 (n = 5)
0 4 8 1216 24 36 48 60 72 84 96
20
Novel TDF- and ABC-Sparing ART
Strategies
Slide credit: clinicaloptions.com
Study Initial or Switch
From Suppr. ART
N Regimen Results
GARDEL[1]
Initial 426 LPV/RTV + 3TC Similar efficacy as LPV/RTV + 2 NRTIs
NEAT001/
ANRS143[2]
Initial 805 DRV/RTV + RAL Similar efficacy as DRV/RTV + FTC/TDF
PADDLE[3,4]
Initial 20 DTG + 3TC Small study; encouraging efficacy
ACTG 5353[5]
Initial 120 DTG + 3TC Encouraging efficacy; 1 pt w/resistance at VF
LAMIDOL[6]
Switch 110 DTG + 3TC Encouraging efficacy
SALT[7]
Switch 286 ATV/RTV + 3TC Similar efficacy as ATV/RTV + 2 NRTIs
ATLAS-M[8]
Switch 266 ATV/RTV + 3TC Noninferior and superior efficacy vs ATV/RTV + 2
NRTIs
OLE[9]
Switch 250 LPV/RTV + 3TC Similar efficacy as cont. standard ART
ANDES[10]
Initial 145 DRV/RTV + 3TC Similar efficacy as DRV/RTV + 3TC/TDF at interim
NA[11]
Switch 48 DRV/RTV + 3TC Small study; encouraging efficacy
LATTE[12]
Switch 243 CAB + RPV Similar efficacy as cont. standard ART
SWORD[13]
Switch 1024 DTG + RPV Similar efficacy as cont. standard ART
LATTE-2[14]
Induction-
Maintenance
309 Ind: CAB+ABC/3TC PO;
Mainten: LA CAB + LA
RPV IM Q4W or Q8W
Similar efficacy as cont. PO CAB + ABC/3TC;
injection-site reactions frequent but high pt
satisfaction
References in slidenotes.
CrCl Cutoffs for Single-Tablet Regimens
1. EVG/COBI/FTC/TDF [package insert]. 2. EFV/FTC/TDF [package
insert]. 3. RPV/FTC/TDF [package insert]. 4. DTG/ABC/3TC
[package insert. 5. EVG/COBI/FTC/TAF [package insert]. 6.
RPV/FTC/TAF [package insert].
Single-Tablet Regimen FDA Approved for Pts With CrCl,
mL/min
EVG/COBI/FTC/TDF[1]
≥ 70
EFV/FTC/TDF[2]
≥ 50
RPV/FTC/TDF[3]
≥ 50
DTG/ABC/3TC[4]
≥ 50
EVG/COBI/FTC/TAF[5]
≥ 30
RPV/FTC/TAF[6]
≥ 30
Slide credit: clinicaloptions.com
Antiretroviral Contraindicated Titrate Dose No Dose Adjustment
EFV Atorvastatin
Simvastatin
Pravastatin
Pitavastatin
Rosuvastatin
RPV Atorvastatin
Pitavastatin
ATV/RTV Lovastatin
Simvastatin
Atorvastatin
Pravastatin
Rosuvastatin
Pitavastatin
ATV/COBI Atorvastatin
Lovastatin
Simvastatin
Pravastatin
Rosuvastatin
Pitavastatin
DRV/RTV
DRV/COBI
Lovastatin
Simvastatin
Atorvastatin
Pravastatin
Rosuvastatin
Pitavastatin
EVG/COBI/FTC/TAF Lovastatin
Simvastatin
Atorvastatin
Rosuvastatin
EVG/COBI/FTC/TDF Lovastatin
Simvastatin
Atorvastatin
Rosuvastatin
DTG or RAL All
Drug–Drug Interactions With First-line
ART and Lipid-Lowering Therapy
DHHS Adult Guidelines. October 2017. US Food and Drug Administration. Slide credit: clinicaloptions.com
Statin Dosing in the Setting of ART
Dubé MP. Lipid management. 2015. p. 241-255. Slide credit: clinicaloptions.com
PI- or COBI-Containing Regimens
High-Intensity Statin Moderate-Intensity Statin Low-Intensity Statin
Atorvastatin 20 mg Atorvastatin 10 mg Pravastatin 10-20 mg
Rosuvastatin 10-20 mg Rosuvastatin 5 mg Fluvastatin 20-40 mg
Pravastatin 40-80 mg* Pitavastatin 1 mg
Pitavastatin 2-4 mg
Simvastatin and lovastatin are contraindicated for pts receiving a PI or COBI
*With darunavir, reduce pravastatin to 20-40 mg
NNRTI-, RAL-, or DTG-Containing Regimens
High-Intensity Statin Moderate-Intensity Statin Low-Intensity Statin
Atorvastatin 40-80 mg Atorvastatin 10-20 mg Pravastatin 10-20 mg
Rosuvastatin 20 mg Rosuvastatin 10 mg Fluvastatin 20-40 mg
Pravastatin 40-80 mg Pitavastatin 1 mg
Pitavastatin 2-4 mg Lovastatin 20 mg
Lovastatin 40 mg Simvastatin 10 mg
Simvastatin 20-40 mg
All doses daily.
Case 1: Issues for Discussion
 Should MJ be prescribed a statin?
 What ART should MJ be prescribed?
 What else can be done to reduce MJ’s risk for new or
worsening comorbidities?
Slide credit: clinicaloptions.com
 56-yr-old black woman diagnosed with HIV and initiated ART 6 yrs ago
 Fell out of care and discontinued ART 1 yr ago, now returned to care
 HIV-1 RNA 21,000 copies/mL; CD4+ cell count 322 cells/mm3
 HIV GT WT, but was viremic on boosted PI + FTC/TDF 1 yr ago
 HBV immune, HCV negative, HLA-B*5701 negative
 Serum creatinine 1.08 mg/dL (MDRD > 60 mL/min/1.73 m2
); dyslipidemia
 Receiving enalapril for HTN
 Smokes 1 pack cigarettes per day for > 30 yrs
Case 1: Additional Considerations
 Smoking cessation
 Is aspirin indicated?
 Is MJ’s blood pressure adequately controlled?
 Should you order a chest CT?
 Avoiding future comorbidities[1]
– Colonoscopy
– PAP smear
– Mammogram
1. Aberg JA, et al. Clin Infect Dis. 2014;58:1-10. Slide credit: clinicaloptions.com
– DXA
– Vaccinations
– STI screening
Case 1: Take-home Points
 CVD is prevalent among HIV-infected individuals,
many of whom have major CVD risk factors
 The ASCVD risk calculator replaces the Framingham
estimation and has established a threshold for statin
initiation as a 7.5% 10-yr risk
 Different ARV agents have varying affects on lipids
and drug interactions with medications used to treat
CVD
 The potential impact of ARV agents on CVD risk must
be considered when selecting HIV therapy
Slide credit: clinicaloptions.com
Case 2: Patient on Long-term
Suppressive ART With Low BMD
Identified After Fracture
Case 2: Pt on Long-term ART With Low
BMD Identified After Fracture
 FR is a 62-yr-old HIV-infected man with suppressed
HIV-1 RNA for > 5 yrs
– Nadir CD4+ cell count 105 cells/mm3
– Baseline HIV genotype: wild type
– Initial and current ART: DRV/RTV + FTC/TDF, initiated
during a clinical trial that has since ended
– HCV negative and HBV immune
– HLA-B*5701 negative
Slide credit: clinicaloptions.com
Case 2: Medical History and Current
Medications
 Past medical history and current medications
– COPD
– No current comedications
 Former smoker
 Currently drinks 2-3 beers nightly
Slide credit: clinicaloptions.com
Case 2: Current Presentation
 At routine visit 4 wks ago, presented in arm cast for wrist
fracture sustained from fall on icy walkway
– HIV-1 RNA < 20 copies/mL
– CD4+ cell count 525 cells/mm3
– Serum creatinine 0.9 mg/dL (MDRD > 60 mL/min/1.73 m2
)
– Urinalysis: trace protein
– ASCVD 10-yr risk: < 7.5%
– Vitamin D (25OH): 22 ng/mL (normal: > 30 ng/mL)
– DXA T-scores: L-spine, -2.4; femoral neck, -2.6; hip, -2.5
Slide credit: clinicaloptions.com
Case 2: Issues for Discussion
 Should FR’s ART be modified?
 Is any further assessment or intervention needed for
FR?
 62-yr-old HIV-infected man with suppressed HIV-1 RNA for > 5 yrs
 Initial and current ART: DRV/RTV + FTC/TDF
 HIV baseline GT WT, HBV immune, HCV negative, HLA-B*5701 negative
 Recent fracture
 DXA T-scores: L-spine, -2.6; femoral neck, -2.7; hip, -2.6
 Former smoker, current daily alcohol use
 COPD
 Serum creatinine 1.2 mg/dL (MDRD > 60 mL/min/1.73 m2
)
Slide credit: clinicaloptions.com
Case 2 Question: How would you manage
FR’s low BMD?
A. Continue DRV/RTV + FTC/TDF and start calcium/vitamin D ±
bisphosphonate
B. Switch FTC/TDF to ABC/3TC
C. Switch FTC/TDF to FTC/TAF
D. Switch DRV/RTV to boosted ATV or an INSTI or RPV
E. Switch to DTG + RPV
F. Something else
 62-yr-old HIV-infected man with suppressed HIV-1 RNA for > 5 yrs; initial/current ART:
DRV/RTV + FTC/TDF; HIV baseline GT WT, HBV immune, HCV negative, HLA-B*5701
negative; low BMD and recent fracture; former smoker; current daily alcohol use; COPD;
serum Cr 1.2 mg/dL (MDRD > 60 mL/min/1.73 m2
)
Do HIV-Positive Pts Have Increased Risk
of Bone Loss and Fractures?
 Meta-analysis: HIV-positive pts had 6.4-fold increased risk of low BMD
and 3.7-fold increased risk of osteoporosis[1]
 8525 HIV-infected pts compared with 2,208,792 uninfected pts in
Partners HealthCare System, 1996-2008[2]
Women Men
1. Brown TT, et al. AIDS. 2006;20:2165-2174.
2. Triant V, et al. J Clin Endocrinol Metab. 2008;93:3499-3504.
Age (Yrs)
7.0
6.0
5.0
4.0
3.0
2.0
1.0
0
FracturePrevalence/
100Persons
30-39 40-49 50-59 60-69 70-79
P = .002
(overall comparison)
HIV
Non-HIV
HIV
Non-HIV
Age (Yrs)
7.0
6.0
5.0
4.0
3.0
2.0
1.0
0
FracturePrevalence/
100Persons
20-29 30-39 40-49 50-59 60-69
P < .0001
(overall comparison)
Slide credit: clinicaloptions.com
Hypothetical Evolution of Bone Mass in
HIV Infection
Orwoll ES, et al. Endocrinol Rev. 1995;16:87-116.
Arpadi S, et al. J AIDS Clin Res. 2008;16:117-121. Slide credit: clinicaloptions.com
BMD
0
Age (Yrs)
Men
Women
HIV
Infection
ART
Initiation
HIV+ men
HIV+ women
3010 20 40 50 60 70 80
1.2
1.0
0.8
0.6
0.4
0.2
0
START Substudy: BMD Changes With
Immediate vs Deferred ART Over 3 Yrs
 START trial: increased serious
AIDS and non-AIDS events with
deferred (until CD4+ ≤ 350
cells/mm³) vs immediate ART
– HR: 0.43; P < .001
 Substudy included 193 pts in
early ART arm and 204 pts in
deferred ART arm with f/u
 Greater BMD loss with
immediate vs deferred ART
– Estimated mean difference:
-1.5% for hip (P < .001); -1.6%
for spine (P < .001)
 Osteoporosis incidence similar
between arms (P = .27)
Hoy JF, et al. EACS 2015. Abstract ADRLH-62.
ChangeFromBL(%)ChangeFromBL(%)
Total Hip BMD
0
-1
-2
-3
-4
-5
0 12 24 36
Immediate ART
Deferred ART
Total Spine BMD
0
-1
-2
-3
-4
-5
0 12 24 36
Mos From Randomization
Slide credit: clinicaloptions.com
A5224s: Mean Percent Change in Hip and
Spine BMD
 Substudy of A5202:
FTC/TDF vs ABC/3TC
with either ATV/RTV vs
EFV
 N = 269
– 85% male, 47% white,
median age: 38 yrs
 Significantly greater
spine and hip BMD loss
with FTC/TDF vs
ABC/3TC
 Significantly greater
BMD loss in spine but
not hip with ATV/RTV vs
EFV
McComsey GA, et al. J Infect Dis. 2011;203:1791-1801.
Wk
Wk
P = .004
P = .024
P = .61
Slide credit: clinicaloptions.com
SpineBMD%
ChangeFromWk0
0
-1
-2
-3
-4
-5
0
1
NRTI Component:
1° Analysis
24 48 96 144 192
FTC/TDF
ABC/3TC
HipBMD%
ChangeFromWk0
0
-1
-2
-3
-4
-5
0
1
24 48 96 144 192
FTC/TDF
ABC/3TC
0
-1
-2
-3
-4
-5
0
1
NNRTI/PI Component:
2° Analysis
24 48 96 144 192
EFV
ATV/RTV
P = .035
0
-1
-2
-3
-4
-5
0
1
24 48 96 144 192
EFV
ATV/RTV
A5260s Substudy of ACTG 5257: BMD
Loss With RAL vs Boosted PIs + FTC/TDF
 A5257: phase III trial in which
treatment-naive pts with HIV-1
RNA ≥ 1000 copies/mL were
randomized to:
– RAL + FTC/TDF (n = 603)
– ATV/RTV + FTC/TDF (n = 605)
– DRV/RTV + FTC/TDF (n = 601)
 In A5260s metabolic substudy (N =
328), all arms associated with
significant loss of BMD through Wk
96 (P < .001)
 At hip and spine, similar loss of
BMD in the PI arms
– Significantly greater loss in
combined PI arms than in RAL arm
ATV/RTV + FTC/TDF
DRV/RTV + FTC/TDF
Combined PI arms
RAL + FTC/TDF
-5
-4
0
-3
-2
-1
-3.9
-3.4
-3.7
-2.4
-1.8
-4.0
-3.8
-3.6
P = .36
Total Hip Lumbar Spine
P = .005
P = .42
P < .001
Brown TT, et al. J Infect Dis. 2015;212:1241-1249. Slide credit: clinicaloptions.com
TAF vs TDF in Studies 104/111: BMD
Changes by Age in Treatment-Naive Pts
 Wk 144 data: persistence of greater spine and hip BMD loss with TDF
vs TAF; 6 discontinuations for bone AEs in TDF arm vs 0 in TAF arm[2]
1. Wohl D, et al. EACS 2015. Abstract 1091.
2. Arribas JR, et al. CROI 2017. Abstract 453. Slide credit: clinicaloptions.com
Difference
E/C/F/TAF
E/C/F/TDF
Mean%ChangeinBMDFromBaseline
DifferenceinLSM(%)
Hip[1]
Spine[1]
18-25 Yrs of Age
All Ages All Ages0
-1
-2
-3
-4 0
2
1
3
4
0
-1
-2
-3
-4 0
2
1
3
4
0 24 48 72 96
0
-1
-2
-3
-4 0
2
1
3
4
18-25 Yrs of Age0
-1
-2
-3
-4 0
2
1
3
4
0 24 48 72 96
Wk Wk
Slide credit: clinicaloptions.com
Raffi F, et al. HIV Glasgow 2016. Abstract O125.
Raffi F, et al. J Acquir Immune Defic Syndr. 2017;75:226-231.
Switch From TDF- to TAF-Containing ART
in Suppressed Pts: Wk 96 BMD Changes
 FTC/TAF noninferior to FTC/TDF for HIV-1 RNA < 50
c/mL at Wks 48 and 96
MeanChangeinBMD(%)
P < .001
3
2
1
0
-1
2.2
-0.2
Wk
BL 24 48 72 96
FTC/TAF
FTC/TDF
321
320
310
310
300
306
294
297
287
292
P < .001
3
2
1
0
-1
1.9
-0.3
Wk
BL 24 48 72 96
FTC/TAF
FTC/TDF
321
317
309
305
300
303
293
296
288
289
Spine Hip
Switch to EVG/COBI/FTC/TAF in
Virologically Suppressed Women
 WAVES: randomized, double-blind phase III trial comparing
EVG/COBI/FTC/TDF vs ATV/RTV + FTC/TDF in 575 treatment-naive
women[1]
 Women completing ATV/RTV arm in WAVES rerandomized[2]
Slide credit: clinicaloptions.com
1. Squires K, et al. Lancet HIV. 2016;3:e410-e420.
2. Hodder S, et al. CROI 2017. Abstract 443.
Switch to EVG/COBI/FTC/TAF
(n = 159)
Continue ATV/RTV + FTC/TDF
(n = 53)
Virologically suppressed
women who completed
48 wks of ATV/RTV +
FTC/TDF in WAVES (N
= 212)
Wk 48
Wk 48 HIV-1
RNA < 50 c/mL
(FDA Snapshot)
94%
87%
Mean BMD Change From
Baseline to Wk 48, %
Switch to
EVG/COBI/FTC/TAF
Continued ATV/RTV +
FTC/TDF
P Value
Spine 2.8 0 < .001
Hip 2.1 1.3 .29
ZEST: Zoledronic Acid vs TDF Switch to
Improve BMD in Pts on TDF-Based ART
 Randomized study comparing switch to non–TDF-based ART vs continuing
TDF-based ART + zoledronic acid* (5 mg IV at Mos 0 and 12) in pts with low
BMD†
and virologic suppression on TDF-based ART (N = 87)
Hoy J, et al. IAS 2017. Abstract WEAB0106LB. Slide credit: clinicaloptions.com
*Calcium and vitamin D supplementation (as indicated) were also provided. †
T-score ≤ -1.0 at spine (L1-
L4) or left femoral neck by DXA. ‡
Primary endpoint, ITT population.
Lumbar Spine BMD Change at 24 Mos‡
ChangeinBMDFromBL(%)
8
6
4
2
0
0 12 24
Mos
6.1
7.4
2.9 2.9
P < .001
P < .001
Continue TDF + ZOL
TDF switch
Outcome,
24 Mos
Continue
TDF + ZOL
(n = 43)
TDF
Switch
(n = 42)
P
Value
Femoral neck
BMD ∆ from
BL, %
4.1 2.1 .03
Total hip BMD
∆ from BL, %
4.6 2.6 .009
Fractures
(events), %
2 17 .03
Fractures
(pts), %
2 10 .20
Mean eGFR ∆ -6.0 3.3 .003
SWORD 1 & 2: Switch From Suppressive
ART to DTG + RPV Dual Therapy
 Randomized, open-label, multicenter phase III trials
 HIV-1 RNA < 50 c/mL at Wk 48 (primary endpoint; ITT-E snapshot)
– 95% in both arms; Wk 48 treatment difference showed noninferiority of
switch: -0.2% (95% CI: -3.0% to 2.5%)
 Significantly greater improvement in bone turnover markers from
baseline to Wk 48 in switch arm
 Coformulated DTG/RPV FDA approved Nov. 2017 as switch regimen
for pts with viral suppression and no resistance or previous tx failure
Walmsley S, et al. IDWeek 2017. Abstract 1382.
Llibre JM, et al. CROI 2017. Abstract 44LB. Slide credit: clinicaloptions.com
Switch to DTG + RPV
(n = 513)
Continue Baseline ART
(n = 511)
Pts with HIV-1 RNA < 50 c/mL
for ≥ 12 mos while receiving first or
second ART regimen with 2 NRTIs
+ INSTI, NNRTI, or PI; no previous
VF; HBV negative
(N = 1024)
Wk 52
Switch to DTG + RPV
Continue DTG + RPV
Case 2: Issues for Discussion
 Should FR’s ART be modified?
 Is any further assessment or intervention needed for
FR?
 62-yr-old HIV-infected man with suppressed HIV-1 RNA for > 5 yrs
 Initial and current ART: DRV/RTV + FTC/TDF
 HIV baseline GT WT, HBV immune, HCV negative, HLA-B*5701 negative
 Recent fracture
 DXA T-scores: L-spine, -2.6; femoral neck, -2.7; hip, -2.6
 Former smoker; current daily alcohol use
 COPD
Slide credit: clinicaloptions.com
Recommendations for Evaluation and
Management of Bone Disease in HIV
 DXA should be performed
– Men ≥ 50 yrs of age
– Postmenopausal women
– People with a history of
fragility fracture
– Pts receiving chronic
glucocorticoid treatment
– People at high risk for
falls
 If low bone mineral density
is detected
– Assess for secondary
causes (eg, vitamin D
deficiency, hypothyroid,
hypogonadism)
– Calcium and vitamin D
supplementation
– Bisphosphonate may be
indicated if osteoporosis
detected
– Avoidance of TDF
Brown TT, et al. Clin Infect Dis. 2015;60:1242-1251. Slide credit: clinicaloptions.com
Calculating Fracture Risk: FRAX Tool
 Developed by WHO to evaluate fracture risk, based on cohort
study data from North America, Europe, Asia, Australia[1]
– Integrates clinical risk factors (smoking status, alcohol
consumption, rheumatoid arthritis) as well as BMD at the femoral
neck, age, and sex
– Provides 10-yr probability of hip fracture and 10-yr probability of
major osteoporotic fracture (clinical fracture in spine, forearm, hip,
or shoulder)
– Online risk calculators and paper charts for white, black, Hispanic,
and Asian populations in the United States and for other countries
available at: http://www.shef.ac.uk/FRAX/
 FRAX underestimated fracture risk in study of HIV+ and HIV-
US veterans[2]
1. FRAX tool. http://www.shef.ac.uk/FRAX/.
2. Yin MT, et al. J Acquir Immune Defic Syndr. 2016;72:513-520. Slide credit: clinicaloptions.com
Case 2: Take-home Points
 Low bone density is common in pts living with HIV
 Traditional and HIV-related factors, including ART,
can lead to bone density loss
 There are recommendations for bone density
screening for HIV-infected men and women
 DXA scanning and the FRAX calculator can be used
to assess bone health
 Secondary causes of low BMD should be sought and,
if found, addressed
Slide credit: clinicaloptions.com
Case 3: Patient With HIV/GT1b
HCV Coinfection
Case 3: Pt With HIV/GT1b HCV Coinfection
 JA is a 46-yr-old woman diagnosed with HIV and HCV
coinfection 10 mos ago
– Active IDU (oxymorphone, heroin)
– No comedications
 She was screened for HIV when a small HIV outbreak was
detected among IDUs in her community
Slide credit: clinicaloptions.com
 HCV parameters at
diagnosis
– HCV genotype 1b
– HCV RNA 1.4 million IU/mL
 HIV parameters at
diagnosis
– HIV-1 RNA 57,000
copies/mL
– CD4+ cell count 584
cells/mm3
– HIV genotype: WT-only
virus
Case 3: Additional Laboratory Values
 Serum creatinine 0.86 mg/dL
(MDRD > 60 mL/min/1.73 m2
)
 AST 78 IU/L, ALT 119 IU/L, total bilirubin 0.8 mg/dL
 Hct 43%, platelets 146,000/mm3
 Albumin 3.7 g/dL
 HBsAb positive, HAV Ab positive
 FibroSure prediction: F3 fibrosis
Slide credit: clinicaloptions.com
Case 3: Current Presentation
 She has been receiving ATV/COBI + FTC/TDF for
6 mos
– HIV-1 RNA < 40 copies/mL
– CD4+ cell count 682 cells/mm3
 She is still injecting but has been accessing clean
needles through a local advocacy group
 She wants to start HCV therapy
Slide credit: clinicaloptions.com
Case 3: Issues for Discussion
 How best to manage JA’s HIV and HCV coinfection
‒ What HCV therapy should be prescribed?
‒ Does JA’s ART need to be modified?
 46-yr-old HIV/GT1b HCV–coinfected woman with suppressed HIV-1 RNA on
ATV/COBI + FTC/TDF
 F3 liver fibrosis
 HIV baseline GT WT, HBV immune
 HCV RNA 1.4 million IU/mL
 Serum creatinine 0.9 mg/dL (MDRD > 60 mL/min/1.73 m2
)
 Current substance abuse (heroin)
 No comedications
Slide credit: clinicaloptions.com
NA-ACCORD (1996-2010): ESLD and
Modern ART in HIV/HCV Coinfection
 Risk for ESLD* in modern
ART era (adjusted IR per
1000 person-yrs)†
– HIV only: 1.26
– HIV/HCV: 6.86
– HIV/HBV: 7.50
– HIV/HCV/HBV: 16.98
 No clear reduction in ESLD
risk over the 3 ART eras
ESLD Incidence Rates
HIV
Incidence(per1000Person-Yrs)
HIV/HCV HIV/HBV HIV/HCV/
HBV
Early ART era (1996-2000, n = 10,395)
Middle ART era (2001-2005, n = 21,188)
Modern ART era (2006-2010, n = 22,472)
Klein MB, et al. CROI 2015. Abstract 638.
Klein MB, et al. Clin Infect Dis. 2016;63:1160-1167.
Infection
Slide credit: clinicaloptions.com
*ESLD as indicated by events such as ascites,
spontaneous bacterial peritonitis, bleeding varices,
encephalopathy, hepatocellular carcinoma.
†
Adjusted for age, sex, race, CD4+ cell count, HIV-1
RNA.
Trials of HCV Therapy in HIV/HCV-
Coinfected Pts
1. Naggie S, et al. N Engl J Med. 2015;373:705-713. 2. Wyles DL, et
al. N Engl J Med. 2015;373:714-725. 3. Wyles D, et al. Clin Infect Dis.
2017;65:6-12. 4. Rockstroh J, et al. IAS 2017. Abstract MOAB0303.
Study Population HCV Regimens
SVR12,
%
ION-4[1] N = 335; GT1 (98%)
or 4
LDV/SOF 12 wks 96
ALLY-2[2] N = 151; GT1 (83%),
2, 3, or 4
Tx naive:
SOF + DCV 12 wks
SOF + DCV 8 wks
Tx exp’d: SOF + DCV 12 wks
97
76
98
ASTRAL-5[3]
N = 106; GT1-4 SOF/VEL 12 wks 95
EXPEDITION-2[4]
N = 153; GT1-6
GLE/PIB for 8 wks without cirrhosis
or 12 wks with cirrhosis
98
Slide credit: clinicaloptions.com
AASLD Guidance on HIV/HCV DDIs
DCV + SOF EBR/GZR GLE/PIB LDV/SOF SOF/VEL SOF/VEL/VOX
ATV + RTV ≈ Χ Χ ≈ ≈ Χ
DRV + RTV √ Χ Χ ≈ ≈ ≈
Tipranavir + RTV Χ Χ Χ Χ Χ Χ
EFV ≈ Χ Χ √ Χ Χ
ETR ≈ Χ √ Χ
RPV √ √ √ √ √ √
DTG or RAL √ √ √ √ √ √
EVG + COBI ≈ Χ ≈ ≈ ≈ ≈
3TC/ABC √ √ √ √ √ √
TAF √ √ √ √ √ √
TDF √ √ √ ≈ ≈ ≈
Slide credit: clinicaloptions.comAASLD/IDSA HCV Guidance. September 2017.
No clinically significant
interaction expected
Potential interaction may require adjustment to
dosage, timing of administration, or monitoring
Do not coadminister
Case 3: HCV Therapy Selection
 Her insurance will only cover elbasvir/grazoprevir for
12 wks for the treatment of her HCV infection
Slide credit: clinicaloptions.com
 46-yr-old HIV/GT1b HCV–coinfected woman with F3 liver fibrosis; HIV baseline GT WT,
HBV immune; serum Cr 0.9 mg/dL (MDRD > 60 mL/min/1.73 m2
)
 Current substance abuse (heroin); no comedications
Case 3 Question: How would you manage JA’s ART
regimen (ATV/COBI + FTC/TDF) to avoid potential
detrimental DDIs when treating her HCV infection
with EBR/GZR?
A. Stay on current regimen
B. Switch to DRV + RTV + FTC/TAF
C. Switch to EVG/COBI/FTC/TAF
D. Switch to DTG/ABC/3TC
E. Switch to RAL QD + FTC/TAF
F. Something else
Recommendations for EBR/GZR Use in
HIV/HCV Coinfection
Slide credit: clinicaloptions.com
1. Rockstroh JK, et al. Lancet HIV. 2015;2:e319-e327. 2. EBR/GZR
[package insert]. 3. Yeh WW, et al. International Workshop Clin Pharm
HIV/Hep Therapy 2015. Abstract 63. 4. AASLD/IDSA. HCV guidance.
September 2017.
EBR/GZR[4]
 ARVs without clinically significant interactions: 3TC, ABC, DTG,
ENF, FTC, RAL, RPV, and TDF
 Should NOT be used with COBI, EFV, ETR, NVP, or any HIV PI
 Phase III study of EBR/GZR in HIV/HCV coinfection allowed
3TC, ABC, FTC, TDF, RAL, DTG, RPV[1]
 EBR/GZR contraindicated or not recommended with all boosted
ART regimens[2]
– Marked increase in GZR exposure (potential for hepatotoxicity)
 EFV results in a 80% reduction in GZR exposure, 50%
decrease in EBR exposure[3]
Case 3: Take-home Points
 There has been an increase in injecting drug use in
the United States and, with it, transmission of HCV
 New direct-acting antiviral regimens for treating HCV
infection are potent but differ in their potential for
drug–drug interactions with ARV agents and other
medications
 Treatment of HIV/HCV coinfection may require
consideration for the modification of HIV therapy
during HCV treatment
Slide credit: clinicaloptions.com
Go Online for More CCO
Coverage of HIV!
Additional slidesets on contemporary management of HIV with expert
faculty commentary
Postconference clinical updates available following CROI, the
International AIDS Conference, and IDWeek
clinicaloptions.com/hiv

More Related Content

What's hot

What's hot (20)

Современное лечение ВИЧ: модификация АРТ у пациентов с вирусной супрессией и ...
Современное лечение ВИЧ: модификация АРТ у пациентов с вирусной супрессией и ...Современное лечение ВИЧ: модификация АРТ у пациентов с вирусной супрессией и ...
Современное лечение ВИЧ: модификация АРТ у пациентов с вирусной супрессией и ...
 
Современное лечение ВИЧ : АРТ как профилактика.Contemporary Management of HIV...
Современное лечение ВИЧ : АРТ как профилактика.Contemporary Management of HIV...Современное лечение ВИЧ : АРТ как профилактика.Contemporary Management of HIV...
Современное лечение ВИЧ : АРТ как профилактика.Contemporary Management of HIV...
 
Современное лечение ВИЧ: индивидуализация стартовой АРТ /Contemporary Manage...
Современное лечение ВИЧ:  индивидуализация стартовой АРТ /Contemporary Manage...Современное лечение ВИЧ:  индивидуализация стартовой АРТ /Contemporary Manage...
Современное лечение ВИЧ: индивидуализация стартовой АРТ /Contemporary Manage...
 
Determining Candidacy and Strategies for ART Modification.2019
Determining Candidacy and Strategies for ART Modification.2019Determining Candidacy and Strategies for ART Modification.2019
Determining Candidacy and Strategies for ART Modification.2019
 
Модификация схем АРТ у пациентов с вирусной супрессией и после вирусологичес...
Модификация схем АРТ у пациентов с  вирусной супрессией и после вирусологичес...Модификация схем АРТ у пациентов с  вирусной супрессией и после вирусологичес...
Модификация схем АРТ у пациентов с вирусной супрессией и после вирусологичес...
 
Современное лечение ВИЧ: новые подходы к оптимизации АРТ/Contemporary Managem...
Современное лечение ВИЧ: новые подходы к оптимизации АРТ/Contemporary Managem...Современное лечение ВИЧ: новые подходы к оптимизации АРТ/Contemporary Managem...
Современное лечение ВИЧ: новые подходы к оптимизации АРТ/Contemporary Managem...
 
Современное лечение ВИЧ: лечение ВИЧ у пациентов с вирусными гепатитами.Conte...
Современное лечение ВИЧ: лечение ВИЧ у пациентов с вирусными гепатитами.Conte...Современное лечение ВИЧ: лечение ВИЧ у пациентов с вирусными гепатитами.Conte...
Современное лечение ВИЧ: лечение ВИЧ у пациентов с вирусными гепатитами.Conte...
 
Антиретровирусные средства и хроническая болезнь почек.Exposure to antiretrov...
Антиретровирусные средства и хроническая болезнь почек.Exposure to antiretrov...Антиретровирусные средства и хроническая болезнь почек.Exposure to antiretrov...
Антиретровирусные средства и хроническая болезнь почек.Exposure to antiretrov...
 
Современное лечение ВИЧ: новые парадигмы в АРТ / Contemporary Management of H...
Современное лечение ВИЧ: новые парадигмы в АРТ / Contemporary Management of H...Современное лечение ВИЧ: новые парадигмы в АРТ / Contemporary Management of H...
Современное лечение ВИЧ: новые парадигмы в АРТ / Contemporary Management of H...
 
HIV Alert:ART Considerations for Aging Patients.2018
HIV Alert:ART Considerations for Aging Patients.2018HIV Alert:ART Considerations for Aging Patients.2018
HIV Alert:ART Considerations for Aging Patients.2018
 
Antiretroviral Therapy Update 2016
Antiretroviral Therapy Update 2016Antiretroviral Therapy Update 2016
Antiretroviral Therapy Update 2016
 
Начало АРТ впервые.Наилучшая практика.Best Practices in Antiretroviral Therap...
Начало АРТ впервые.Наилучшая практика.Best Practices in Antiretroviral Therap...Начало АРТ впервые.Наилучшая практика.Best Practices in Antiretroviral Therap...
Начало АРТ впервые.Наилучшая практика.Best Practices in Antiretroviral Therap...
 
Сравнение режимов лечения ВИЧ в разрезе различных клинических сценариев.ART...
Сравнение  режимов лечения ВИЧ в  разрезе различных клинических сценариев.ART...Сравнение  режимов лечения ВИЧ в  разрезе различных клинических сценариев.ART...
Сравнение режимов лечения ВИЧ в разрезе различных клинических сценариев.ART...
 
Современное лечение ВИЧ: когда начинать, чем начинать. Contemporary Managemen...
Современное лечение ВИЧ: когда начинать, чем начинать. Contemporary Managemen...Современное лечение ВИЧ: когда начинать, чем начинать. Contemporary Managemen...
Современное лечение ВИЧ: когда начинать, чем начинать. Contemporary Managemen...
 
Современное лечение ВИЧ.Усилить или не усилить : преимущества и недостатки бу...
Современное лечение ВИЧ.Усилить или не усилить : преимущества и недостатки бу...Современное лечение ВИЧ.Усилить или не усилить : преимущества и недостатки бу...
Современное лечение ВИЧ.Усилить или не усилить : преимущества и недостатки бу...
 
Confronting the Challenges of HIV Care in an Aging Population.2019
Confronting the Challenges of HIV Care in an Aging Population.2019Confronting the Challenges of HIV Care in an Aging Population.2019
Confronting the Challenges of HIV Care in an Aging Population.2019
 
Ключевые решения в лечении ВИЧ: оптимизация стратегии лечения для пациентов с...
Ключевые решения в лечении ВИЧ: оптимизация стратегии лечения для пациентов с...Ключевые решения в лечении ВИЧ: оптимизация стратегии лечения для пациентов с...
Ключевые решения в лечении ВИЧ: оптимизация стратегии лечения для пациентов с...
 
Integrating Recent Data When Selecting First-line Antiretroviral Therapy.2015...
Integrating Recent Data When Selecting First-line Antiretroviral Therapy.2015...Integrating Recent Data When Selecting First-line Antiretroviral Therapy.2015...
Integrating Recent Data When Selecting First-line Antiretroviral Therapy.2015...
 
АРТ в 2016-2017 гг: неизменная потребность в индивидуализации лечения для улу...
АРТ в 2016-2017 гг: неизменная потребность в индивидуализации лечения для улу...АРТ в 2016-2017 гг: неизменная потребность в индивидуализации лечения для улу...
АРТ в 2016-2017 гг: неизменная потребность в индивидуализации лечения для улу...
 
Should Integrase Inhibitors Be Your First Choice When Starting HIV Therapy- E...
Should Integrase Inhibitors Be Your First Choice When Starting HIV Therapy- E...Should Integrase Inhibitors Be Your First Choice When Starting HIV Therapy- E...
Should Integrase Inhibitors Be Your First Choice When Starting HIV Therapy- E...
 

Similar to Contemporary Management of HIV.How Common Comorbidities Affect ART Management.2018

First and foremost choosing and using first line antiretroviral therapy.2013
First and foremost choosing and using first line antiretroviral therapy.2013First and foremost choosing and using first line antiretroviral therapy.2013
First and foremost choosing and using first line antiretroviral therapy.2013
Hivlife Info
 
Start impaact june 7 2011
Start impaact june 7 2011Start impaact june 7 2011
Start impaact june 7 2011
Phil Boehmer
 
Start impaact june 7 2011
Start impaact june 7 2011Start impaact june 7 2011
Start impaact june 7 2011
Phil Boehmer
 
SYNCing Guidelines- Catanzaro
SYNCing Guidelines- CatanzaroSYNCing Guidelines- Catanzaro
SYNCing Guidelines- Catanzaro
healthhiv
 
D1 Highly Active Antiretroviral Treatment (HAART) DHHS Guidelines 2009 Duffus
D1 Highly Active Antiretroviral Treatment (HAART) DHHS Guidelines 2009 DuffusD1 Highly Active Antiretroviral Treatment (HAART) DHHS Guidelines 2009 Duffus
D1 Highly Active Antiretroviral Treatment (HAART) DHHS Guidelines 2009 Duffus
DSHS
 
Hiv eye update 2013
Hiv eye update 2013Hiv eye update 2013
Hiv eye update 2013
etedaldi
 
C5 Case Study Session of Three Long-Term Survivors with HIV Disease Mondy
C5 Case Study Session of Three Long-Term Survivors with HIV Disease MondyC5 Case Study Session of Three Long-Term Survivors with HIV Disease Mondy
C5 Case Study Session of Three Long-Term Survivors with HIV Disease Mondy
DSHS
 
C6 HIV 201 Armas
C6 HIV 201 ArmasC6 HIV 201 Armas
C6 HIV 201 Armas
DSHS
 

Similar to Contemporary Management of HIV.How Common Comorbidities Affect ART Management.2018 (20)

Key Slides on ART for HIV : Evolving Concepts and Innovative Strategies.2020
Key Slides on ART for HIV : Evolving Concepts and Innovative Strategies.2020Key Slides on ART for HIV : Evolving Concepts and Innovative Strategies.2020
Key Slides on ART for HIV : Evolving Concepts and Innovative Strategies.2020
 
First and foremost choosing and using first line antiretroviral therapy.2013
First and foremost choosing and using first line antiretroviral therapy.2013First and foremost choosing and using first line antiretroviral therapy.2013
First and foremost choosing and using first line antiretroviral therapy.2013
 
First and foremost choosing and using first line antiretroviral therapy.2013
First and foremost choosing and using first line antiretroviral therapy.2013First and foremost choosing and using first line antiretroviral therapy.2013
First and foremost choosing and using first line antiretroviral therapy.2013
 
Современное лечение и профилактика ВИЧ : передовые стратегии лечения у пациен...
Современное лечение и профилактика ВИЧ : передовые стратегии лечения у пациен...Современное лечение и профилактика ВИЧ : передовые стратегии лечения у пациен...
Современное лечение и профилактика ВИЧ : передовые стратегии лечения у пациен...
 
Современное лечение ВИЧ: модификация АРТ у пациентов с вирусологической супре...
Современное лечение ВИЧ: модификация АРТ у пациентов с вирусологической супре...Современное лечение ВИЧ: модификация АРТ у пациентов с вирусологической супре...
Современное лечение ВИЧ: модификация АРТ у пациентов с вирусологической супре...
 
Start impaact june 7 2011
Start impaact june 7 2011Start impaact june 7 2011
Start impaact june 7 2011
 
Start impaact june 7 2011
Start impaact june 7 2011Start impaact june 7 2011
Start impaact june 7 2011
 
Выбор начальной схемы АРТ у пациентов старшего возраста.Choosing and Using F...
Выбор начальной схемы АРТ у  пациентов старшего возраста.Choosing and Using F...Выбор начальной схемы АРТ у  пациентов старшего возраста.Choosing and Using F...
Выбор начальной схемы АРТ у пациентов старшего возраста.Choosing and Using F...
 
Challenging Cases in HIV Management.2014
Challenging Cases in HIV Management.2014 Challenging Cases in HIV Management.2014
Challenging Cases in HIV Management.2014
 
Challenging Cases in HIV Management.2014
Challenging Cases in HIV Management.2014Challenging Cases in HIV Management.2014
Challenging Cases in HIV Management.2014
 
The State of the Art in HIV Cure Research – Hope or Hype: What Does It Mean f...
The State of the Art in HIV Cure Research – Hope or Hype: What Does It Mean f...The State of the Art in HIV Cure Research – Hope or Hype: What Does It Mean f...
The State of the Art in HIV Cure Research – Hope or Hype: What Does It Mean f...
 
SYNCing Guidelines- Catanzaro
SYNCing Guidelines- CatanzaroSYNCing Guidelines- Catanzaro
SYNCing Guidelines- Catanzaro
 
D1 Highly Active Antiretroviral Treatment (HAART) DHHS Guidelines 2009 Duffus
D1 Highly Active Antiretroviral Treatment (HAART) DHHS Guidelines 2009 DuffusD1 Highly Active Antiretroviral Treatment (HAART) DHHS Guidelines 2009 Duffus
D1 Highly Active Antiretroviral Treatment (HAART) DHHS Guidelines 2009 Duffus
 
Hiv eye update 2013
Hiv eye update 2013Hiv eye update 2013
Hiv eye update 2013
 
C5 Case Study Session of Three Long-Term Survivors with HIV Disease Mondy
C5 Case Study Session of Three Long-Term Survivors with HIV Disease MondyC5 Case Study Session of Three Long-Term Survivors with HIV Disease Mondy
C5 Case Study Session of Three Long-Term Survivors with HIV Disease Mondy
 
C6 HIV 201 Armas
C6 HIV 201 ArmasC6 HIV 201 Armas
C6 HIV 201 Armas
 
Highlights of AIDS 2014 .CCO Official Conference Coverage of the 20th Interna...
Highlights of AIDS 2014 .CCO Official Conference Coverage of the 20th Interna...Highlights of AIDS 2014 .CCO Official Conference Coverage of the 20th Interna...
Highlights of AIDS 2014 .CCO Official Conference Coverage of the 20th Interna...
 
What Drives the Excess Risk? Determinants of Chronic Complications in HIV
What Drives the Excess Risk? Determinants of Chronic Complications in HIVWhat Drives the Excess Risk? Determinants of Chronic Complications in HIV
What Drives the Excess Risk? Determinants of Chronic Complications in HIV
 
Solid Organ Transplantation and HIV
Solid Organ Transplantation and HIVSolid Organ Transplantation and HIV
Solid Organ Transplantation and HIV
 
journal.pone.0006828.PDF
journal.pone.0006828.PDFjournal.pone.0006828.PDF
journal.pone.0006828.PDF
 

More from hivlifeinfo

Гиперлипопротеидемия(а) как опасное генетически обусловленное нарушение липид...
Гиперлипопротеидемия(а) как опасное генетически обусловленное нарушение липид...Гиперлипопротеидемия(а) как опасное генетически обусловленное нарушение липид...
Гиперлипопротеидемия(а) как опасное генетически обусловленное нарушение липид...
hivlifeinfo
 
Физическая активность и физические тренировки как метод профилактики сердечно...
Физическая активность и физические тренировки как метод профилактики сердечно...Физическая активность и физические тренировки как метод профилактики сердечно...
Физическая активность и физические тренировки как метод профилактики сердечно...
hivlifeinfo
 
Свобода интернета 2018: делегирование репрессий.Доклад Международной Агоры
Свобода интернета 2018: делегирование репрессий.Доклад Международной АгорыСвобода интернета 2018: делегирование репрессий.Доклад Международной Агоры
Свобода интернета 2018: делегирование репрессий.Доклад Международной Агоры
hivlifeinfo
 

More from hivlifeinfo (19)

Дискуссии о здоровом старении с ВИЧ /Key Slides on Healthy Aging With HIV.2022
Дискуссии о здоровом старении с ВИЧ /Key Slides on Healthy Aging With HIV.2022Дискуссии о здоровом старении с ВИЧ /Key Slides on Healthy Aging With HIV.2022
Дискуссии о здоровом старении с ВИЧ /Key Slides on Healthy Aging With HIV.2022
 
Основы ведения АРТ у многократно леченных пациентов 2022 / Foundations of ART...
Основы ведения АРТ у многократно леченных пациентов 2022 / Foundations of ART...Основы ведения АРТ у многократно леченных пациентов 2022 / Foundations of ART...
Основы ведения АРТ у многократно леченных пациентов 2022 / Foundations of ART...
 
Ключевые слайды по индивидуальному выбору АРТ / Key Slides on Individualized ...
Ключевые слайды по индивидуальному выбору АРТ / Key Slides on Individualized ...Ключевые слайды по индивидуальному выбору АРТ / Key Slides on Individualized ...
Ключевые слайды по индивидуальному выбору АРТ / Key Slides on Individualized ...
 
Clinical Impact of New Data From AIDS 2020
Clinical Impact of New Data From AIDS 2020Clinical Impact of New Data From AIDS 2020
Clinical Impact of New Data From AIDS 2020
 
Слайдсет о новом в лечении ВИЧ.Key Slides on What’s Hot in HIV Treatment.2020
Слайдсет о новом в лечении ВИЧ.Key Slides on What’s Hot in HIV Treatment.2020 Слайдсет о новом в лечении ВИЧ.Key Slides on What’s Hot in HIV Treatment.2020
Слайдсет о новом в лечении ВИЧ.Key Slides on What’s Hot in HIV Treatment.2020
 
Гиперлипопротеидемия(а) как опасное генетически обусловленное нарушение липид...
Гиперлипопротеидемия(а) как опасное генетически обусловленное нарушение липид...Гиперлипопротеидемия(а) как опасное генетически обусловленное нарушение липид...
Гиперлипопротеидемия(а) как опасное генетически обусловленное нарушение липид...
 
Физическая активность и физические тренировки как метод профилактики сердечно...
Физическая активность и физические тренировки как метод профилактики сердечно...Физическая активность и физические тренировки как метод профилактики сердечно...
Физическая активность и физические тренировки как метод профилактики сердечно...
 
Общие принципы ведения пациентов с ХБП
Общие принципы ведения пациентов с ХБПОбщие принципы ведения пациентов с ХБП
Общие принципы ведения пациентов с ХБП
 
Симптомы заболеваний почек (краткий клинический анализ)
Симптомы заболеваний почек (краткий клинический анализ)Симптомы заболеваний почек (краткий клинический анализ)
Симптомы заболеваний почек (краткий клинический анализ)
 
Клинические рекомендации «Алгоритмы специализированной медицинской помощи бол...
Клинические рекомендации «Алгоритмы специализированной медицинской помощи бол...Клинические рекомендации «Алгоритмы специализированной медицинской помощи бол...
Клинические рекомендации «Алгоритмы специализированной медицинской помощи бол...
 
Key Slides on Individualizing ART Management Based on Treatment Safety and To...
Key Slides on Individualizing ART Management Based on Treatment Safety and To...Key Slides on Individualizing ART Management Based on Treatment Safety and To...
Key Slides on Individualizing ART Management Based on Treatment Safety and To...
 
Современное лечение ВИЧ.Обобщённые данные с конференции CROI 2020 / Contempor...
Современное лечение ВИЧ.Обобщённые данные с конференции CROI 2020 / Contempor...Современное лечение ВИЧ.Обобщённые данные с конференции CROI 2020 / Contempor...
Современное лечение ВИЧ.Обобщённые данные с конференции CROI 2020 / Contempor...
 
Incorporating New ART Options Into First-line and Switch Strategies for HIV C...
Incorporating New ART Options Into First-line and Switch Strategies for HIV C...Incorporating New ART Options Into First-line and Switch Strategies for HIV C...
Incorporating New ART Options Into First-line and Switch Strategies for HIV C...
 
Свобода интернета 2018: делегирование репрессий.Доклад Международной Агоры
Свобода интернета 2018: делегирование репрессий.Доклад Международной АгорыСвобода интернета 2018: делегирование репрессий.Доклад Международной Агоры
Свобода интернета 2018: делегирование репрессий.Доклад Международной Агоры
 
Современное лечение ВИЧ.Объединенные данные с конференции IAS 2019 / Contemp...
Современное лечение ВИЧ.Объединенные данные с конференции  IAS 2019 / Contemp...Современное лечение ВИЧ.Объединенные данные с конференции  IAS 2019 / Contemp...
Современное лечение ВИЧ.Объединенные данные с конференции IAS 2019 / Contemp...
 
Clinical Impact of New Data From IAS 2019
Clinical Impact of New Data From IAS 2019Clinical Impact of New Data From IAS 2019
Clinical Impact of New Data From IAS 2019
 
Предиабет-определение, риски, подходы к диагностике и профилактике сахарного ...
Предиабет-определение, риски, подходы к диагностике и профилактике сахарного ...Предиабет-определение, риски, подходы к диагностике и профилактике сахарного ...
Предиабет-определение, риски, подходы к диагностике и профилактике сахарного ...
 
Tsepamo: DTG Exposure at Conception Associated With Smaller Increase in Incid...
Tsepamo: DTG Exposure at Conception Associated With Smaller Increase in Incid...Tsepamo: DTG Exposure at Conception Associated With Smaller Increase in Incid...
Tsepamo: DTG Exposure at Conception Associated With Smaller Increase in Incid...
 
Случаи и разногласия по ВИЧ в 2019 году: европейские перспективы / Cases and...
 Случаи и разногласия по ВИЧ в 2019 году: европейские перспективы / Cases and... Случаи и разногласия по ВИЧ в 2019 году: европейские перспективы / Cases and...
Случаи и разногласия по ВИЧ в 2019 году: европейские перспективы / Cases and...
 

Recently uploaded

Hyderabad Call Girls Service ❤️ 7783825323 Independent Escort Service Hyderabad
Hyderabad Call Girls Service ❤️ 7783825323 Independent Escort Service HyderabadHyderabad Call Girls Service ❤️ 7783825323 Independent Escort Service Hyderabad
Hyderabad Call Girls Service ❤️ 7783825323 Independent Escort Service Hyderabad
Sheetaleventcompany
 
Call Girls Pune Just Call 9142599079 Top Class Call Girl Service Available
Call Girls Pune Just Call 9142599079 Top Class Call Girl Service AvailableCall Girls Pune Just Call 9142599079 Top Class Call Girl Service Available
Call Girls Pune Just Call 9142599079 Top Class Call Girl Service Available
Sheetaleventcompany
 
Pune Call Girl Service 📞9xx000xx09📞Just Call Divya📲 Call Girl In Pune No💰Adva...
Pune Call Girl Service 📞9xx000xx09📞Just Call Divya📲 Call Girl In Pune No💰Adva...Pune Call Girl Service 📞9xx000xx09📞Just Call Divya📲 Call Girl In Pune No💰Adva...
Pune Call Girl Service 📞9xx000xx09📞Just Call Divya📲 Call Girl In Pune No💰Adva...
Sheetaleventcompany
 
💚Call Girls In Amritsar 💯Anvi 📲🔝8725944379🔝Amritsar Call Girl No💰Advance Cash...
💚Call Girls In Amritsar 💯Anvi 📲🔝8725944379🔝Amritsar Call Girl No💰Advance Cash...💚Call Girls In Amritsar 💯Anvi 📲🔝8725944379🔝Amritsar Call Girl No💰Advance Cash...
💚Call Girls In Amritsar 💯Anvi 📲🔝8725944379🔝Amritsar Call Girl No💰Advance Cash...
Sheetaleventcompany
 
❤️ Chandigarh Call Girls☎️98151-579OO☎️ Call Girl service in Chandigarh ☎️ Ch...
❤️ Chandigarh Call Girls☎️98151-579OO☎️ Call Girl service in Chandigarh ☎️ Ch...❤️ Chandigarh Call Girls☎️98151-579OO☎️ Call Girl service in Chandigarh ☎️ Ch...
❤️ Chandigarh Call Girls☎️98151-579OO☎️ Call Girl service in Chandigarh ☎️ Ch...
Rashmi Entertainment
 

Recently uploaded (20)

Race Course Road } Book Call Girls in Bangalore | Whatsapp No 6378878445 VIP ...
Race Course Road } Book Call Girls in Bangalore | Whatsapp No 6378878445 VIP ...Race Course Road } Book Call Girls in Bangalore | Whatsapp No 6378878445 VIP ...
Race Course Road } Book Call Girls in Bangalore | Whatsapp No 6378878445 VIP ...
 
Call Girl In Chandigarh 📞9809698092📞 Just📲 Call Inaaya Chandigarh Call Girls ...
Call Girl In Chandigarh 📞9809698092📞 Just📲 Call Inaaya Chandigarh Call Girls ...Call Girl In Chandigarh 📞9809698092📞 Just📲 Call Inaaya Chandigarh Call Girls ...
Call Girl In Chandigarh 📞9809698092📞 Just📲 Call Inaaya Chandigarh Call Girls ...
 
Circulatory Shock, types and stages, compensatory mechanisms
Circulatory Shock, types and stages, compensatory mechanismsCirculatory Shock, types and stages, compensatory mechanisms
Circulatory Shock, types and stages, compensatory mechanisms
 
Hyderabad Call Girls Service ❤️ 7783825323 Independent Escort Service Hyderabad
Hyderabad Call Girls Service ❤️ 7783825323 Independent Escort Service HyderabadHyderabad Call Girls Service ❤️ 7783825323 Independent Escort Service Hyderabad
Hyderabad Call Girls Service ❤️ 7783825323 Independent Escort Service Hyderabad
 
Call Girls Pune Just Call 9142599079 Top Class Call Girl Service Available
Call Girls Pune Just Call 9142599079 Top Class Call Girl Service AvailableCall Girls Pune Just Call 9142599079 Top Class Call Girl Service Available
Call Girls Pune Just Call 9142599079 Top Class Call Girl Service Available
 
Lucknow Call Girls Service { 9984666624 } ❤️VVIP ROCKY Call Girl in Lucknow U...
Lucknow Call Girls Service { 9984666624 } ❤️VVIP ROCKY Call Girl in Lucknow U...Lucknow Call Girls Service { 9984666624 } ❤️VVIP ROCKY Call Girl in Lucknow U...
Lucknow Call Girls Service { 9984666624 } ❤️VVIP ROCKY Call Girl in Lucknow U...
 
(RIYA)🎄Airhostess Call Girl Jaipur Call Now 8445551418 Premium Collection Of ...
(RIYA)🎄Airhostess Call Girl Jaipur Call Now 8445551418 Premium Collection Of ...(RIYA)🎄Airhostess Call Girl Jaipur Call Now 8445551418 Premium Collection Of ...
(RIYA)🎄Airhostess Call Girl Jaipur Call Now 8445551418 Premium Collection Of ...
 
Bhawanipatna Call Girls 📞9332606886 Call Girls in Bhawanipatna Escorts servic...
Bhawanipatna Call Girls 📞9332606886 Call Girls in Bhawanipatna Escorts servic...Bhawanipatna Call Girls 📞9332606886 Call Girls in Bhawanipatna Escorts servic...
Bhawanipatna Call Girls 📞9332606886 Call Girls in Bhawanipatna Escorts servic...
 
Chandigarh Call Girls Service ❤️🍑 9809698092 👄🫦Independent Escort Service Cha...
Chandigarh Call Girls Service ❤️🍑 9809698092 👄🫦Independent Escort Service Cha...Chandigarh Call Girls Service ❤️🍑 9809698092 👄🫦Independent Escort Service Cha...
Chandigarh Call Girls Service ❤️🍑 9809698092 👄🫦Independent Escort Service Cha...
 
💞 Safe And Secure Call Girls Coimbatore🧿 6378878445 🧿 High Class Coimbatore C...
💞 Safe And Secure Call Girls Coimbatore🧿 6378878445 🧿 High Class Coimbatore C...💞 Safe And Secure Call Girls Coimbatore🧿 6378878445 🧿 High Class Coimbatore C...
💞 Safe And Secure Call Girls Coimbatore🧿 6378878445 🧿 High Class Coimbatore C...
 
Chennai ❣️ Call Girl 6378878445 Call Girls in Chennai Escort service book now
Chennai ❣️ Call Girl 6378878445 Call Girls in Chennai Escort service book nowChennai ❣️ Call Girl 6378878445 Call Girls in Chennai Escort service book now
Chennai ❣️ Call Girl 6378878445 Call Girls in Chennai Escort service book now
 
ANATOMY AND PHYSIOLOGY OF RESPIRATORY SYSTEM.pptx
ANATOMY AND PHYSIOLOGY OF RESPIRATORY SYSTEM.pptxANATOMY AND PHYSIOLOGY OF RESPIRATORY SYSTEM.pptx
ANATOMY AND PHYSIOLOGY OF RESPIRATORY SYSTEM.pptx
 
Pune Call Girl Service 📞9xx000xx09📞Just Call Divya📲 Call Girl In Pune No💰Adva...
Pune Call Girl Service 📞9xx000xx09📞Just Call Divya📲 Call Girl In Pune No💰Adva...Pune Call Girl Service 📞9xx000xx09📞Just Call Divya📲 Call Girl In Pune No💰Adva...
Pune Call Girl Service 📞9xx000xx09📞Just Call Divya📲 Call Girl In Pune No💰Adva...
 
💚Call Girls In Amritsar 💯Anvi 📲🔝8725944379🔝Amritsar Call Girl No💰Advance Cash...
💚Call Girls In Amritsar 💯Anvi 📲🔝8725944379🔝Amritsar Call Girl No💰Advance Cash...💚Call Girls In Amritsar 💯Anvi 📲🔝8725944379🔝Amritsar Call Girl No💰Advance Cash...
💚Call Girls In Amritsar 💯Anvi 📲🔝8725944379🔝Amritsar Call Girl No💰Advance Cash...
 
Call Girls Kathua Just Call 8250077686 Top Class Call Girl Service Available
Call Girls Kathua Just Call 8250077686 Top Class Call Girl Service AvailableCall Girls Kathua Just Call 8250077686 Top Class Call Girl Service Available
Call Girls Kathua Just Call 8250077686 Top Class Call Girl Service Available
 
Call Girls in Lucknow Just Call 👉👉 8875999948 Top Class Call Girl Service Ava...
Call Girls in Lucknow Just Call 👉👉 8875999948 Top Class Call Girl Service Ava...Call Girls in Lucknow Just Call 👉👉 8875999948 Top Class Call Girl Service Ava...
Call Girls in Lucknow Just Call 👉👉 8875999948 Top Class Call Girl Service Ava...
 
Call Girls Bangalore - 450+ Call Girl Cash Payment 💯Call Us 🔝 6378878445 🔝 💃 ...
Call Girls Bangalore - 450+ Call Girl Cash Payment 💯Call Us 🔝 6378878445 🔝 💃 ...Call Girls Bangalore - 450+ Call Girl Cash Payment 💯Call Us 🔝 6378878445 🔝 💃 ...
Call Girls Bangalore - 450+ Call Girl Cash Payment 💯Call Us 🔝 6378878445 🔝 💃 ...
 
❤️ Chandigarh Call Girls☎️98151-579OO☎️ Call Girl service in Chandigarh ☎️ Ch...
❤️ Chandigarh Call Girls☎️98151-579OO☎️ Call Girl service in Chandigarh ☎️ Ch...❤️ Chandigarh Call Girls☎️98151-579OO☎️ Call Girl service in Chandigarh ☎️ Ch...
❤️ Chandigarh Call Girls☎️98151-579OO☎️ Call Girl service in Chandigarh ☎️ Ch...
 
Call Girls Rishikesh Just Call 9667172968 Top Class Call Girl Service Available
Call Girls Rishikesh Just Call 9667172968 Top Class Call Girl Service AvailableCall Girls Rishikesh Just Call 9667172968 Top Class Call Girl Service Available
Call Girls Rishikesh Just Call 9667172968 Top Class Call Girl Service Available
 
ANATOMY AND PHYSIOLOGY OF REPRODUCTIVE SYSTEM.pptx
ANATOMY AND PHYSIOLOGY OF REPRODUCTIVE SYSTEM.pptxANATOMY AND PHYSIOLOGY OF REPRODUCTIVE SYSTEM.pptx
ANATOMY AND PHYSIOLOGY OF REPRODUCTIVE SYSTEM.pptx
 

Contemporary Management of HIV.How Common Comorbidities Affect ART Management.2018

  • 1. Contemporary Management of HIV: How Common Comorbidities Affect ART Management This program is supported by an independent educational grant from ViiV Healthcare.
  • 2.  Please feel free to use, update, and share some or all of these slides in your noncommercial presentations to colleagues or patients  When using our slides, please retain the source attribution:  These slides may not be published, posted online, or used in commercial presentations without permission. Please contact permissions@clinicaloptions.com for details Slide credit: clinicaloptions.com About These Slides
  • 3. Program Director and Core Faculty Program Chair Joseph J. Eron, Jr., MD Professor of Medicine and Epidemiology University of North Carolina School of Medicine Director, AIDS Clinical Trials Unit University of North Carolina Chapel Hill, North Carolina David A. Wohl, MD Professor of Medicine School of Medicine Site Leader, AIDS Clinical Trials Unit-Chapel Hill University of North Carolina at Chapel Hill Director, North Carolina AIDS Training and Education Center Chapel Hill, North Carolina Co-Director, HIV Services North Carolina Department of Correction Raleigh, North Carolina
  • 4. Faculty Disclosure Information Joseph J. Eron, Jr., MD, has disclosed that he has received consulting fees from Gilead Sciences, Janssen, Merck, and ViiV and funds for research support from AbbVie, Gilead Sciences, Janssen, and ViiV. David A. Wohl, MD, has disclosed that he has received consulting fees from Bristol-Myers Squibb, Gilead Sciences, Janssen, and ViiV and funds for research support from Gilead Sciences.
  • 5. Peer Review Disclosure Barry S. Zingman, MD Medical Director, AIDS Center Clinical Director, Infectious Diseases, Moses Division Professor of Clinical Medicine, Albert Einstein College of Medicine Montefiore Medical Center The University Hospital for Albert Einstein College of Medicine Barry S. Zingman, MD, has no real or apparent conflicts of interest to report.
  • 6. Program Overview  Comanaging CVD risk and ART selection in middle- aged HIV-infected pts  Managing low bone mineral density and fracture risk in older pts on long-term suppressive ART  Managing concomitant ART and HCV therapy in HIV/HCV-coinfected pts
  • 7. Case 1: Patient With CVD Risk
  • 8. Case 1: Pt With CVD Risk  MJ is a 56-yr-old black woman diagnosed with HIV infection 6 yrs ago  She fell out of care 1 yr ago and stopped ART  1 mo ago, she returned to care and was started on enalapril for hypertension  Smokes 1 pack cigarettes per day for > 30 yrs Slide credit: clinicaloptions.com
  • 9. Case 1: HIV/ART History  Laboratory values at the time of HIV diagnosis – CD4+ cell count 486 cells/mm3 – HIV-1 RNA 12,000 copies/mL  Started on boosted PI + FTC/TDF – HIV-1 RNA was suppressed except for occasional blips  Prior to falling out of care, her last CD4+ cell count was 744 cells/mm3 and HIV-1 RNA was 410 copies/mL  Stopped ART when her father passed away and she took on care of her 80-yr-old mother; her mother has since moved to a nursing home allowing her to address her own healthcare needs Slide credit: clinicaloptions.com
  • 10.  Current presentation today – BP 142/88 mm Hg – BMI 27 – Urinalysis: trace protein – Toxicology screen: negative – HLA-B*5701 negative – TC 207 mg/dL, HDL 42 mg/dL, LDL 130 mg/dL, TG 176 mg/dL Case 1: Recent History and Current Presentation  Returned to care 1 mo ago – HIV-1 RNA 21,000 copies/mL – CD4+ cell count 322 cells/mm3 – HCV negative; HBV immune – Serum creatinine 1.13 mg/dL (MDRD > 60 mL/min/1.73 m2 ) – HIV genotype: wild-type virus Slide credit: clinicaloptions.com
  • 11. Case 1: Issues for Discussion  Should MJ be prescribed a statin?  What ART should MJ be prescribed?  What else can be done to reduce MJ’s risk for new or worsening comorbidities? Slide credit: clinicaloptions.com  56-yr-old black woman diagnosed with HIV and initiated ART 6 yrs ago  Fell out of care and discontinued ART 1 yr ago, now returned to care  HIV-1 RNA 21,000 copies/mL; CD4+ cell count 322 cells/mm3  HIV GT WT, but was viremic on boosted PI + FTC/TDF 1 yr ago  HBV immune, HCV negative, HLA-B*5701 negative  Serum creatinine 1.13 mg/dL (MDRD > 60 mL/min/1.73 m2 ); dyslipidemia  Receiving enalapril for HTN  Smokes 1 pack cigarettes per day for > 30 yrs
  • 12. Declines in MI, Stroke in United States: Kaiser Permanente CA Cohort After adjusting for stroke risk factors: No increased risk in HIV+ with recent CD4+ cell count ≥ 500 cells/mm3 or recent HIV-1 RNA < 500 c/mL Increased risk in HIV+ with recent CD4+ cell count < 500 cells/mm3 or recent HIV-1 RNA ≥ 500 c/mL Recent reduced MI rates for HIV+ likely due to: CVD risk reduction, lipid-friendly ART, reduced immunodeficiency 1. Klein DB, et al. CROI 2014. Abstract 737. 2. Klein DB, et al. Clin Infect Dis. 2015;60:1278-1280. 3. Marcus JL, et al. CROI 2014. Abstract 741. 4. Marcus JL, et al. AIDS. 2014;28:1911-1919. 200 150 100 50 1996-1999 2000-2003 2004-2007 2008-2009 2010-2011 250 0 Stroke Rates by HIV Status and Yr[3,4] Cases/100,000PYs 400 MIs/100,000PYs 300 200 100 0 1996-99 2000-03 2004-07 2008-09 2010-11 HIV+ HIV- MI Rates Over Time by HIV Status[1,2] HIV+ HIV- Slide credit: clinicaloptions.com
  • 13. Guesses? http://tools.acc.org/ASCVD-Risk-Estimator/ Case 1: ASCVD Risk Estimator Any Slide credit: clinicaloptions.com
  • 14. http://tools.acc.org/ASCVD-Risk-Estimator/ Case 1: ASCVD Risk Estimator Slide credit: clinicaloptions.com
  • 15. http://tools.acc.org/ASCVD-Risk-Estimator/ ASCVD Risk Estimator: Recommendations Adults 40-75 yrs of age with LDL 70-189 mg/dL with no diabetes and estimated 10-yr ASCVD risk ≥ 7.5% should be treated with moderate- to high-intensity statin therapy (I A) Slide credit: clinicaloptions.com
  • 16.  CVD risk scores calculated with data from 2006-2009 for pts in Partners HealthCare System Cohort[1]  Analysis of HOPS cohort (n = 2283)found FRS accurately estimated CVD risk but AHA/ACC and D:A:D underestimated risk[2] 1. Regan S, et al. CROI 2015. Abstract 751. 2. Thompson-Paul AM, et al. Clin Infect Dis. 2016;63:1508-1516. 5-Yr Predicted Rate (%) Framingham Risk Score[1] 5-YrEventRate(%) 5-YrEventRate(%) ACC/AHACVD Risk Calculator[1] 5-Yr Predicted Rate (%) Observed Predicted Observed Predicted CVD Outcomes Underestimated in HIV- Positive Pts by Risk Calculators n = 2270 n = 2152 25 20 15 10 5 0 < 2.5 2.5-4.9 5.0-7.4 7.5-9.9 25 20 15 10 5 0 < 2.5 2.5-4.9 5.0-7.4 7.5-9.9 Slide credit: clinicaloptions.com
  • 17. Case 1: Issues for Discussion  Should MJ be prescribed a statin?  What ART should MJ be prescribed?  What else can be done to reduce MJ’s risk for new or worsening comorbidities? Slide credit: clinicaloptions.com  56-yr-old black woman diagnosed with HIV and initiated ART 6 yrs ago  Fell out of care and discontinued ART 1 yr ago, now returned to care  HIV-1 RNA 21,000 copies/mL; CD4+ cell count 322 cells/mm3  HIV GT WT, but was viremic on boosted PI + FTC/TDF 1 yr ago  HBV immune, HCV negative, HLA-B*5701 negative  Serum creatinine 1.13 mg/dL (MDRD > 60 mL/min/1.73 m2 ); dyslipidemia  Receiving enalapril for HTN  Smokes 1 pack cigarettes per day for > 30 yrs  ASCVD 10-yr risk: 18%
  • 18. ART and Effects on Lipids TDF ABCTAF RAL DTG ATV/RTV or ATV/COBI DRV/RTV or DRV/COBI EVG/COBI EFVRPV ETV Slide credit: clinicaloptions.com
  • 19. Slide credit: clinicaloptions.com Gatell JM et al. IAS 2017. Abstract TUAB0102. ClinicalTrials.gov. NCT02098837. NEAT 022: Switch From Boosted PI to DTG in Suppressed Pts With High CV Risk  International, randomized, open-label phase IV study – Primary endpoints at Wk 48: proportion with HIV RNA < 50 copies/mL (ITT), change in total plasma cholesterol  Baseline NRTI backbones: FTC/TDF, 64.8%; ABC/3TC, 31.3% Pts with stable HIV-1 RNA < 50 c/mL on PI/RTV + 2 NRTIs, high CV risk,* no resistance mutations, no VF (N = 415) Immediate switch to DTG + 2 NRTIs† (n = 205) Continue PI/RTV + 2 NRTIs† (n = 210) Deferred switch to DTG + 2 NRTIs† Wk 48 Wk 96 *> 50 yrs of age and/or Framingham risk score > 10% at 10 yrs. † NRTIs to remain the same throughout study.
  • 20.  Switching to DTG noninferior to continuing boosted PI through Wk 48 Slide credit: clinicaloptions.comGatell JM, et al. IAS 2017. Abstract TUAB0102. NEAT 022: Key Findings  Switching to DTG assoc. with improved lipid profile vs cont. boosted PI through Wk 48  No emergent resistance in pts with VF  No significant differences in grade 3/4 AEs, serious AEs, AE-related d/c Virologic Success* Virologic Nonresponse No Virologic Data ITTPopulation(%) Treatment difference: -2.1% (95% CI: -6.6% to 2.4%) 4.94.4 100 80 60 40 20 0 93 95 2 0.5 DTG + 2 NRTIs PI/RTV + 2 NRTIs DTG + 2 NRTIs PI/RTV + 2 NRTIs 0.7 -8.7 -11.3 0.5 4.2 2.0 1.1 2.5 0.4 -18.4 -7.7 -7.0 TC Non– HDL -C TG LDL- C HDL- C TC/ HDL Ratio P < .001 P < .001 P < .001 P < .001 P < .001 P = .286 MeanChange FromBLtoWk48(%) *HIV-1 RNA < 50 copies/mL. 5 -5 -15 10 0 -10 -20 -25
  • 21. Case 1: Issues for Discussion  Should MJ be prescribed a statin?  What ART should MJ be prescribed? – What if MJ’s serum creatinine were 1.9 mg/dL and MDRD were 42 mL/min/1.73 m2 ?  What else can be done to reduce MJ’s risk for new or worsening comorbidities? Slide credit: clinicaloptions.com  HIV-1 RNA 21,000 copies/mL; CD4+ cell count 322 cells/mm3  HIV GT WT, but was viremic on boosted PI + FTC/TDF 1 yr ago  HBV immune, HCV negative, HLA-B*5701 negative  Serum creatinine 1.9 mg/dL (MDRD 42 mL/min/1.73 m2 ); dyslipidemia  Smokes 1 pack cigarettes per day for > 30 yrs  ASCVD 10-yr risk: 18%
  • 22. Case 1 Question: Would you prescribe an ABC-based regimen? A. Yes B. No  HIV-1 RNA 21,000 copies/mL; CD4+ cell count 322 cells/mm3  HIV GT WT, but was viremic on boosted PI + FTC/TDF 1 yr ago  HBV immune, HCV negative, HLA-B*5701 negative  Serum creatinine 1.9 mg/dL (MDRD 42 mL/min/1.73 m2 ); dyslipidemia  Smokes 1 pack of cigarettes per day for > 30 yrs  ASCVD 10-yr risk: 18%
  • 23. Potential Association Between ABC Use and CVD Event Risk: Conflicting Data  Multiple studies have identified increased risk of MI or overall CVD events with ABC use, including large cohort studies, RCTs, and case-control studies[1-10] – Range of risk estimates across studies: 1.3- to 4.3-fold increase  However, several studies have also demonstrated a lack of increased CVD risk with ABC use, including large cohort studies, a meta-analysis of RCTs, and a case-control study[10-14]  DHHS: consider avoiding ABC in pts at elevated risk for CVD[15] References in slidenotes. Slide credit: clinicaloptions.com
  • 24. TAF in Pts With Renal Insufficiency  Open-label trial of 242 virologically suppressed pts with stable eGFRCG 30-69 mL/min switched from TDF and non-TDF regimens to E/C/F/TAF[1] – eGFR remained stable through Wk 144 analysis[2] 1. Post FA, et al. CROI 2016. Abstract 680. 2. Podzamczer D, et al. IAS 2017. Abstract MOPEB0288. Slide credit: clinicaloptions.com MedianeGFR (mL/min/1.73m2 ) Wks eGFR (CKD-EPI, Cystatin C)100 80 60 40 0 Baseline eGFR, mL/min > 60 (n = 163) 51-60 (n = 39) 41-50 (n = 20) 31-40 (n = 14) ≤ 30 (n = 5) 0 4 8 1216 24 36 48 60 72 84 96 20
  • 25. Novel TDF- and ABC-Sparing ART Strategies Slide credit: clinicaloptions.com Study Initial or Switch From Suppr. ART N Regimen Results GARDEL[1] Initial 426 LPV/RTV + 3TC Similar efficacy as LPV/RTV + 2 NRTIs NEAT001/ ANRS143[2] Initial 805 DRV/RTV + RAL Similar efficacy as DRV/RTV + FTC/TDF PADDLE[3,4] Initial 20 DTG + 3TC Small study; encouraging efficacy ACTG 5353[5] Initial 120 DTG + 3TC Encouraging efficacy; 1 pt w/resistance at VF LAMIDOL[6] Switch 110 DTG + 3TC Encouraging efficacy SALT[7] Switch 286 ATV/RTV + 3TC Similar efficacy as ATV/RTV + 2 NRTIs ATLAS-M[8] Switch 266 ATV/RTV + 3TC Noninferior and superior efficacy vs ATV/RTV + 2 NRTIs OLE[9] Switch 250 LPV/RTV + 3TC Similar efficacy as cont. standard ART ANDES[10] Initial 145 DRV/RTV + 3TC Similar efficacy as DRV/RTV + 3TC/TDF at interim NA[11] Switch 48 DRV/RTV + 3TC Small study; encouraging efficacy LATTE[12] Switch 243 CAB + RPV Similar efficacy as cont. standard ART SWORD[13] Switch 1024 DTG + RPV Similar efficacy as cont. standard ART LATTE-2[14] Induction- Maintenance 309 Ind: CAB+ABC/3TC PO; Mainten: LA CAB + LA RPV IM Q4W or Q8W Similar efficacy as cont. PO CAB + ABC/3TC; injection-site reactions frequent but high pt satisfaction References in slidenotes.
  • 26. CrCl Cutoffs for Single-Tablet Regimens 1. EVG/COBI/FTC/TDF [package insert]. 2. EFV/FTC/TDF [package insert]. 3. RPV/FTC/TDF [package insert]. 4. DTG/ABC/3TC [package insert. 5. EVG/COBI/FTC/TAF [package insert]. 6. RPV/FTC/TAF [package insert]. Single-Tablet Regimen FDA Approved for Pts With CrCl, mL/min EVG/COBI/FTC/TDF[1] ≥ 70 EFV/FTC/TDF[2] ≥ 50 RPV/FTC/TDF[3] ≥ 50 DTG/ABC/3TC[4] ≥ 50 EVG/COBI/FTC/TAF[5] ≥ 30 RPV/FTC/TAF[6] ≥ 30 Slide credit: clinicaloptions.com
  • 27. Antiretroviral Contraindicated Titrate Dose No Dose Adjustment EFV Atorvastatin Simvastatin Pravastatin Pitavastatin Rosuvastatin RPV Atorvastatin Pitavastatin ATV/RTV Lovastatin Simvastatin Atorvastatin Pravastatin Rosuvastatin Pitavastatin ATV/COBI Atorvastatin Lovastatin Simvastatin Pravastatin Rosuvastatin Pitavastatin DRV/RTV DRV/COBI Lovastatin Simvastatin Atorvastatin Pravastatin Rosuvastatin Pitavastatin EVG/COBI/FTC/TAF Lovastatin Simvastatin Atorvastatin Rosuvastatin EVG/COBI/FTC/TDF Lovastatin Simvastatin Atorvastatin Rosuvastatin DTG or RAL All Drug–Drug Interactions With First-line ART and Lipid-Lowering Therapy DHHS Adult Guidelines. October 2017. US Food and Drug Administration. Slide credit: clinicaloptions.com
  • 28. Statin Dosing in the Setting of ART Dubé MP. Lipid management. 2015. p. 241-255. Slide credit: clinicaloptions.com PI- or COBI-Containing Regimens High-Intensity Statin Moderate-Intensity Statin Low-Intensity Statin Atorvastatin 20 mg Atorvastatin 10 mg Pravastatin 10-20 mg Rosuvastatin 10-20 mg Rosuvastatin 5 mg Fluvastatin 20-40 mg Pravastatin 40-80 mg* Pitavastatin 1 mg Pitavastatin 2-4 mg Simvastatin and lovastatin are contraindicated for pts receiving a PI or COBI *With darunavir, reduce pravastatin to 20-40 mg NNRTI-, RAL-, or DTG-Containing Regimens High-Intensity Statin Moderate-Intensity Statin Low-Intensity Statin Atorvastatin 40-80 mg Atorvastatin 10-20 mg Pravastatin 10-20 mg Rosuvastatin 20 mg Rosuvastatin 10 mg Fluvastatin 20-40 mg Pravastatin 40-80 mg Pitavastatin 1 mg Pitavastatin 2-4 mg Lovastatin 20 mg Lovastatin 40 mg Simvastatin 10 mg Simvastatin 20-40 mg All doses daily.
  • 29. Case 1: Issues for Discussion  Should MJ be prescribed a statin?  What ART should MJ be prescribed?  What else can be done to reduce MJ’s risk for new or worsening comorbidities? Slide credit: clinicaloptions.com  56-yr-old black woman diagnosed with HIV and initiated ART 6 yrs ago  Fell out of care and discontinued ART 1 yr ago, now returned to care  HIV-1 RNA 21,000 copies/mL; CD4+ cell count 322 cells/mm3  HIV GT WT, but was viremic on boosted PI + FTC/TDF 1 yr ago  HBV immune, HCV negative, HLA-B*5701 negative  Serum creatinine 1.08 mg/dL (MDRD > 60 mL/min/1.73 m2 ); dyslipidemia  Receiving enalapril for HTN  Smokes 1 pack cigarettes per day for > 30 yrs
  • 30. Case 1: Additional Considerations  Smoking cessation  Is aspirin indicated?  Is MJ’s blood pressure adequately controlled?  Should you order a chest CT?  Avoiding future comorbidities[1] – Colonoscopy – PAP smear – Mammogram 1. Aberg JA, et al. Clin Infect Dis. 2014;58:1-10. Slide credit: clinicaloptions.com – DXA – Vaccinations – STI screening
  • 31. Case 1: Take-home Points  CVD is prevalent among HIV-infected individuals, many of whom have major CVD risk factors  The ASCVD risk calculator replaces the Framingham estimation and has established a threshold for statin initiation as a 7.5% 10-yr risk  Different ARV agents have varying affects on lipids and drug interactions with medications used to treat CVD  The potential impact of ARV agents on CVD risk must be considered when selecting HIV therapy Slide credit: clinicaloptions.com
  • 32. Case 2: Patient on Long-term Suppressive ART With Low BMD Identified After Fracture
  • 33. Case 2: Pt on Long-term ART With Low BMD Identified After Fracture  FR is a 62-yr-old HIV-infected man with suppressed HIV-1 RNA for > 5 yrs – Nadir CD4+ cell count 105 cells/mm3 – Baseline HIV genotype: wild type – Initial and current ART: DRV/RTV + FTC/TDF, initiated during a clinical trial that has since ended – HCV negative and HBV immune – HLA-B*5701 negative Slide credit: clinicaloptions.com
  • 34. Case 2: Medical History and Current Medications  Past medical history and current medications – COPD – No current comedications  Former smoker  Currently drinks 2-3 beers nightly Slide credit: clinicaloptions.com
  • 35. Case 2: Current Presentation  At routine visit 4 wks ago, presented in arm cast for wrist fracture sustained from fall on icy walkway – HIV-1 RNA < 20 copies/mL – CD4+ cell count 525 cells/mm3 – Serum creatinine 0.9 mg/dL (MDRD > 60 mL/min/1.73 m2 ) – Urinalysis: trace protein – ASCVD 10-yr risk: < 7.5% – Vitamin D (25OH): 22 ng/mL (normal: > 30 ng/mL) – DXA T-scores: L-spine, -2.4; femoral neck, -2.6; hip, -2.5 Slide credit: clinicaloptions.com
  • 36. Case 2: Issues for Discussion  Should FR’s ART be modified?  Is any further assessment or intervention needed for FR?  62-yr-old HIV-infected man with suppressed HIV-1 RNA for > 5 yrs  Initial and current ART: DRV/RTV + FTC/TDF  HIV baseline GT WT, HBV immune, HCV negative, HLA-B*5701 negative  Recent fracture  DXA T-scores: L-spine, -2.6; femoral neck, -2.7; hip, -2.6  Former smoker, current daily alcohol use  COPD  Serum creatinine 1.2 mg/dL (MDRD > 60 mL/min/1.73 m2 ) Slide credit: clinicaloptions.com
  • 37. Case 2 Question: How would you manage FR’s low BMD? A. Continue DRV/RTV + FTC/TDF and start calcium/vitamin D ± bisphosphonate B. Switch FTC/TDF to ABC/3TC C. Switch FTC/TDF to FTC/TAF D. Switch DRV/RTV to boosted ATV or an INSTI or RPV E. Switch to DTG + RPV F. Something else  62-yr-old HIV-infected man with suppressed HIV-1 RNA for > 5 yrs; initial/current ART: DRV/RTV + FTC/TDF; HIV baseline GT WT, HBV immune, HCV negative, HLA-B*5701 negative; low BMD and recent fracture; former smoker; current daily alcohol use; COPD; serum Cr 1.2 mg/dL (MDRD > 60 mL/min/1.73 m2 )
  • 38. Do HIV-Positive Pts Have Increased Risk of Bone Loss and Fractures?  Meta-analysis: HIV-positive pts had 6.4-fold increased risk of low BMD and 3.7-fold increased risk of osteoporosis[1]  8525 HIV-infected pts compared with 2,208,792 uninfected pts in Partners HealthCare System, 1996-2008[2] Women Men 1. Brown TT, et al. AIDS. 2006;20:2165-2174. 2. Triant V, et al. J Clin Endocrinol Metab. 2008;93:3499-3504. Age (Yrs) 7.0 6.0 5.0 4.0 3.0 2.0 1.0 0 FracturePrevalence/ 100Persons 30-39 40-49 50-59 60-69 70-79 P = .002 (overall comparison) HIV Non-HIV HIV Non-HIV Age (Yrs) 7.0 6.0 5.0 4.0 3.0 2.0 1.0 0 FracturePrevalence/ 100Persons 20-29 30-39 40-49 50-59 60-69 P < .0001 (overall comparison) Slide credit: clinicaloptions.com
  • 39. Hypothetical Evolution of Bone Mass in HIV Infection Orwoll ES, et al. Endocrinol Rev. 1995;16:87-116. Arpadi S, et al. J AIDS Clin Res. 2008;16:117-121. Slide credit: clinicaloptions.com BMD 0 Age (Yrs) Men Women HIV Infection ART Initiation HIV+ men HIV+ women 3010 20 40 50 60 70 80 1.2 1.0 0.8 0.6 0.4 0.2 0
  • 40. START Substudy: BMD Changes With Immediate vs Deferred ART Over 3 Yrs  START trial: increased serious AIDS and non-AIDS events with deferred (until CD4+ ≤ 350 cells/mm³) vs immediate ART – HR: 0.43; P < .001  Substudy included 193 pts in early ART arm and 204 pts in deferred ART arm with f/u  Greater BMD loss with immediate vs deferred ART – Estimated mean difference: -1.5% for hip (P < .001); -1.6% for spine (P < .001)  Osteoporosis incidence similar between arms (P = .27) Hoy JF, et al. EACS 2015. Abstract ADRLH-62. ChangeFromBL(%)ChangeFromBL(%) Total Hip BMD 0 -1 -2 -3 -4 -5 0 12 24 36 Immediate ART Deferred ART Total Spine BMD 0 -1 -2 -3 -4 -5 0 12 24 36 Mos From Randomization Slide credit: clinicaloptions.com
  • 41. A5224s: Mean Percent Change in Hip and Spine BMD  Substudy of A5202: FTC/TDF vs ABC/3TC with either ATV/RTV vs EFV  N = 269 – 85% male, 47% white, median age: 38 yrs  Significantly greater spine and hip BMD loss with FTC/TDF vs ABC/3TC  Significantly greater BMD loss in spine but not hip with ATV/RTV vs EFV McComsey GA, et al. J Infect Dis. 2011;203:1791-1801. Wk Wk P = .004 P = .024 P = .61 Slide credit: clinicaloptions.com SpineBMD% ChangeFromWk0 0 -1 -2 -3 -4 -5 0 1 NRTI Component: 1° Analysis 24 48 96 144 192 FTC/TDF ABC/3TC HipBMD% ChangeFromWk0 0 -1 -2 -3 -4 -5 0 1 24 48 96 144 192 FTC/TDF ABC/3TC 0 -1 -2 -3 -4 -5 0 1 NNRTI/PI Component: 2° Analysis 24 48 96 144 192 EFV ATV/RTV P = .035 0 -1 -2 -3 -4 -5 0 1 24 48 96 144 192 EFV ATV/RTV
  • 42. A5260s Substudy of ACTG 5257: BMD Loss With RAL vs Boosted PIs + FTC/TDF  A5257: phase III trial in which treatment-naive pts with HIV-1 RNA ≥ 1000 copies/mL were randomized to: – RAL + FTC/TDF (n = 603) – ATV/RTV + FTC/TDF (n = 605) – DRV/RTV + FTC/TDF (n = 601)  In A5260s metabolic substudy (N = 328), all arms associated with significant loss of BMD through Wk 96 (P < .001)  At hip and spine, similar loss of BMD in the PI arms – Significantly greater loss in combined PI arms than in RAL arm ATV/RTV + FTC/TDF DRV/RTV + FTC/TDF Combined PI arms RAL + FTC/TDF -5 -4 0 -3 -2 -1 -3.9 -3.4 -3.7 -2.4 -1.8 -4.0 -3.8 -3.6 P = .36 Total Hip Lumbar Spine P = .005 P = .42 P < .001 Brown TT, et al. J Infect Dis. 2015;212:1241-1249. Slide credit: clinicaloptions.com
  • 43. TAF vs TDF in Studies 104/111: BMD Changes by Age in Treatment-Naive Pts  Wk 144 data: persistence of greater spine and hip BMD loss with TDF vs TAF; 6 discontinuations for bone AEs in TDF arm vs 0 in TAF arm[2] 1. Wohl D, et al. EACS 2015. Abstract 1091. 2. Arribas JR, et al. CROI 2017. Abstract 453. Slide credit: clinicaloptions.com Difference E/C/F/TAF E/C/F/TDF Mean%ChangeinBMDFromBaseline DifferenceinLSM(%) Hip[1] Spine[1] 18-25 Yrs of Age All Ages All Ages0 -1 -2 -3 -4 0 2 1 3 4 0 -1 -2 -3 -4 0 2 1 3 4 0 24 48 72 96 0 -1 -2 -3 -4 0 2 1 3 4 18-25 Yrs of Age0 -1 -2 -3 -4 0 2 1 3 4 0 24 48 72 96 Wk Wk
  • 44. Slide credit: clinicaloptions.com Raffi F, et al. HIV Glasgow 2016. Abstract O125. Raffi F, et al. J Acquir Immune Defic Syndr. 2017;75:226-231. Switch From TDF- to TAF-Containing ART in Suppressed Pts: Wk 96 BMD Changes  FTC/TAF noninferior to FTC/TDF for HIV-1 RNA < 50 c/mL at Wks 48 and 96 MeanChangeinBMD(%) P < .001 3 2 1 0 -1 2.2 -0.2 Wk BL 24 48 72 96 FTC/TAF FTC/TDF 321 320 310 310 300 306 294 297 287 292 P < .001 3 2 1 0 -1 1.9 -0.3 Wk BL 24 48 72 96 FTC/TAF FTC/TDF 321 317 309 305 300 303 293 296 288 289 Spine Hip
  • 45. Switch to EVG/COBI/FTC/TAF in Virologically Suppressed Women  WAVES: randomized, double-blind phase III trial comparing EVG/COBI/FTC/TDF vs ATV/RTV + FTC/TDF in 575 treatment-naive women[1]  Women completing ATV/RTV arm in WAVES rerandomized[2] Slide credit: clinicaloptions.com 1. Squires K, et al. Lancet HIV. 2016;3:e410-e420. 2. Hodder S, et al. CROI 2017. Abstract 443. Switch to EVG/COBI/FTC/TAF (n = 159) Continue ATV/RTV + FTC/TDF (n = 53) Virologically suppressed women who completed 48 wks of ATV/RTV + FTC/TDF in WAVES (N = 212) Wk 48 Wk 48 HIV-1 RNA < 50 c/mL (FDA Snapshot) 94% 87% Mean BMD Change From Baseline to Wk 48, % Switch to EVG/COBI/FTC/TAF Continued ATV/RTV + FTC/TDF P Value Spine 2.8 0 < .001 Hip 2.1 1.3 .29
  • 46. ZEST: Zoledronic Acid vs TDF Switch to Improve BMD in Pts on TDF-Based ART  Randomized study comparing switch to non–TDF-based ART vs continuing TDF-based ART + zoledronic acid* (5 mg IV at Mos 0 and 12) in pts with low BMD† and virologic suppression on TDF-based ART (N = 87) Hoy J, et al. IAS 2017. Abstract WEAB0106LB. Slide credit: clinicaloptions.com *Calcium and vitamin D supplementation (as indicated) were also provided. † T-score ≤ -1.0 at spine (L1- L4) or left femoral neck by DXA. ‡ Primary endpoint, ITT population. Lumbar Spine BMD Change at 24 Mos‡ ChangeinBMDFromBL(%) 8 6 4 2 0 0 12 24 Mos 6.1 7.4 2.9 2.9 P < .001 P < .001 Continue TDF + ZOL TDF switch Outcome, 24 Mos Continue TDF + ZOL (n = 43) TDF Switch (n = 42) P Value Femoral neck BMD ∆ from BL, % 4.1 2.1 .03 Total hip BMD ∆ from BL, % 4.6 2.6 .009 Fractures (events), % 2 17 .03 Fractures (pts), % 2 10 .20 Mean eGFR ∆ -6.0 3.3 .003
  • 47. SWORD 1 & 2: Switch From Suppressive ART to DTG + RPV Dual Therapy  Randomized, open-label, multicenter phase III trials  HIV-1 RNA < 50 c/mL at Wk 48 (primary endpoint; ITT-E snapshot) – 95% in both arms; Wk 48 treatment difference showed noninferiority of switch: -0.2% (95% CI: -3.0% to 2.5%)  Significantly greater improvement in bone turnover markers from baseline to Wk 48 in switch arm  Coformulated DTG/RPV FDA approved Nov. 2017 as switch regimen for pts with viral suppression and no resistance or previous tx failure Walmsley S, et al. IDWeek 2017. Abstract 1382. Llibre JM, et al. CROI 2017. Abstract 44LB. Slide credit: clinicaloptions.com Switch to DTG + RPV (n = 513) Continue Baseline ART (n = 511) Pts with HIV-1 RNA < 50 c/mL for ≥ 12 mos while receiving first or second ART regimen with 2 NRTIs + INSTI, NNRTI, or PI; no previous VF; HBV negative (N = 1024) Wk 52 Switch to DTG + RPV Continue DTG + RPV
  • 48. Case 2: Issues for Discussion  Should FR’s ART be modified?  Is any further assessment or intervention needed for FR?  62-yr-old HIV-infected man with suppressed HIV-1 RNA for > 5 yrs  Initial and current ART: DRV/RTV + FTC/TDF  HIV baseline GT WT, HBV immune, HCV negative, HLA-B*5701 negative  Recent fracture  DXA T-scores: L-spine, -2.6; femoral neck, -2.7; hip, -2.6  Former smoker; current daily alcohol use  COPD Slide credit: clinicaloptions.com
  • 49. Recommendations for Evaluation and Management of Bone Disease in HIV  DXA should be performed – Men ≥ 50 yrs of age – Postmenopausal women – People with a history of fragility fracture – Pts receiving chronic glucocorticoid treatment – People at high risk for falls  If low bone mineral density is detected – Assess for secondary causes (eg, vitamin D deficiency, hypothyroid, hypogonadism) – Calcium and vitamin D supplementation – Bisphosphonate may be indicated if osteoporosis detected – Avoidance of TDF Brown TT, et al. Clin Infect Dis. 2015;60:1242-1251. Slide credit: clinicaloptions.com
  • 50. Calculating Fracture Risk: FRAX Tool  Developed by WHO to evaluate fracture risk, based on cohort study data from North America, Europe, Asia, Australia[1] – Integrates clinical risk factors (smoking status, alcohol consumption, rheumatoid arthritis) as well as BMD at the femoral neck, age, and sex – Provides 10-yr probability of hip fracture and 10-yr probability of major osteoporotic fracture (clinical fracture in spine, forearm, hip, or shoulder) – Online risk calculators and paper charts for white, black, Hispanic, and Asian populations in the United States and for other countries available at: http://www.shef.ac.uk/FRAX/  FRAX underestimated fracture risk in study of HIV+ and HIV- US veterans[2] 1. FRAX tool. http://www.shef.ac.uk/FRAX/. 2. Yin MT, et al. J Acquir Immune Defic Syndr. 2016;72:513-520. Slide credit: clinicaloptions.com
  • 51. Case 2: Take-home Points  Low bone density is common in pts living with HIV  Traditional and HIV-related factors, including ART, can lead to bone density loss  There are recommendations for bone density screening for HIV-infected men and women  DXA scanning and the FRAX calculator can be used to assess bone health  Secondary causes of low BMD should be sought and, if found, addressed Slide credit: clinicaloptions.com
  • 52. Case 3: Patient With HIV/GT1b HCV Coinfection
  • 53. Case 3: Pt With HIV/GT1b HCV Coinfection  JA is a 46-yr-old woman diagnosed with HIV and HCV coinfection 10 mos ago – Active IDU (oxymorphone, heroin) – No comedications  She was screened for HIV when a small HIV outbreak was detected among IDUs in her community Slide credit: clinicaloptions.com  HCV parameters at diagnosis – HCV genotype 1b – HCV RNA 1.4 million IU/mL  HIV parameters at diagnosis – HIV-1 RNA 57,000 copies/mL – CD4+ cell count 584 cells/mm3 – HIV genotype: WT-only virus
  • 54. Case 3: Additional Laboratory Values  Serum creatinine 0.86 mg/dL (MDRD > 60 mL/min/1.73 m2 )  AST 78 IU/L, ALT 119 IU/L, total bilirubin 0.8 mg/dL  Hct 43%, platelets 146,000/mm3  Albumin 3.7 g/dL  HBsAb positive, HAV Ab positive  FibroSure prediction: F3 fibrosis Slide credit: clinicaloptions.com
  • 55. Case 3: Current Presentation  She has been receiving ATV/COBI + FTC/TDF for 6 mos – HIV-1 RNA < 40 copies/mL – CD4+ cell count 682 cells/mm3  She is still injecting but has been accessing clean needles through a local advocacy group  She wants to start HCV therapy Slide credit: clinicaloptions.com
  • 56. Case 3: Issues for Discussion  How best to manage JA’s HIV and HCV coinfection ‒ What HCV therapy should be prescribed? ‒ Does JA’s ART need to be modified?  46-yr-old HIV/GT1b HCV–coinfected woman with suppressed HIV-1 RNA on ATV/COBI + FTC/TDF  F3 liver fibrosis  HIV baseline GT WT, HBV immune  HCV RNA 1.4 million IU/mL  Serum creatinine 0.9 mg/dL (MDRD > 60 mL/min/1.73 m2 )  Current substance abuse (heroin)  No comedications Slide credit: clinicaloptions.com
  • 57. NA-ACCORD (1996-2010): ESLD and Modern ART in HIV/HCV Coinfection  Risk for ESLD* in modern ART era (adjusted IR per 1000 person-yrs)† – HIV only: 1.26 – HIV/HCV: 6.86 – HIV/HBV: 7.50 – HIV/HCV/HBV: 16.98  No clear reduction in ESLD risk over the 3 ART eras ESLD Incidence Rates HIV Incidence(per1000Person-Yrs) HIV/HCV HIV/HBV HIV/HCV/ HBV Early ART era (1996-2000, n = 10,395) Middle ART era (2001-2005, n = 21,188) Modern ART era (2006-2010, n = 22,472) Klein MB, et al. CROI 2015. Abstract 638. Klein MB, et al. Clin Infect Dis. 2016;63:1160-1167. Infection Slide credit: clinicaloptions.com *ESLD as indicated by events such as ascites, spontaneous bacterial peritonitis, bleeding varices, encephalopathy, hepatocellular carcinoma. † Adjusted for age, sex, race, CD4+ cell count, HIV-1 RNA.
  • 58. Trials of HCV Therapy in HIV/HCV- Coinfected Pts 1. Naggie S, et al. N Engl J Med. 2015;373:705-713. 2. Wyles DL, et al. N Engl J Med. 2015;373:714-725. 3. Wyles D, et al. Clin Infect Dis. 2017;65:6-12. 4. Rockstroh J, et al. IAS 2017. Abstract MOAB0303. Study Population HCV Regimens SVR12, % ION-4[1] N = 335; GT1 (98%) or 4 LDV/SOF 12 wks 96 ALLY-2[2] N = 151; GT1 (83%), 2, 3, or 4 Tx naive: SOF + DCV 12 wks SOF + DCV 8 wks Tx exp’d: SOF + DCV 12 wks 97 76 98 ASTRAL-5[3] N = 106; GT1-4 SOF/VEL 12 wks 95 EXPEDITION-2[4] N = 153; GT1-6 GLE/PIB for 8 wks without cirrhosis or 12 wks with cirrhosis 98 Slide credit: clinicaloptions.com
  • 59. AASLD Guidance on HIV/HCV DDIs DCV + SOF EBR/GZR GLE/PIB LDV/SOF SOF/VEL SOF/VEL/VOX ATV + RTV ≈ Χ Χ ≈ ≈ Χ DRV + RTV √ Χ Χ ≈ ≈ ≈ Tipranavir + RTV Χ Χ Χ Χ Χ Χ EFV ≈ Χ Χ √ Χ Χ ETR ≈ Χ √ Χ RPV √ √ √ √ √ √ DTG or RAL √ √ √ √ √ √ EVG + COBI ≈ Χ ≈ ≈ ≈ ≈ 3TC/ABC √ √ √ √ √ √ TAF √ √ √ √ √ √ TDF √ √ √ ≈ ≈ ≈ Slide credit: clinicaloptions.comAASLD/IDSA HCV Guidance. September 2017. No clinically significant interaction expected Potential interaction may require adjustment to dosage, timing of administration, or monitoring Do not coadminister
  • 60. Case 3: HCV Therapy Selection  Her insurance will only cover elbasvir/grazoprevir for 12 wks for the treatment of her HCV infection Slide credit: clinicaloptions.com
  • 61.  46-yr-old HIV/GT1b HCV–coinfected woman with F3 liver fibrosis; HIV baseline GT WT, HBV immune; serum Cr 0.9 mg/dL (MDRD > 60 mL/min/1.73 m2 )  Current substance abuse (heroin); no comedications Case 3 Question: How would you manage JA’s ART regimen (ATV/COBI + FTC/TDF) to avoid potential detrimental DDIs when treating her HCV infection with EBR/GZR? A. Stay on current regimen B. Switch to DRV + RTV + FTC/TAF C. Switch to EVG/COBI/FTC/TAF D. Switch to DTG/ABC/3TC E. Switch to RAL QD + FTC/TAF F. Something else
  • 62. Recommendations for EBR/GZR Use in HIV/HCV Coinfection Slide credit: clinicaloptions.com 1. Rockstroh JK, et al. Lancet HIV. 2015;2:e319-e327. 2. EBR/GZR [package insert]. 3. Yeh WW, et al. International Workshop Clin Pharm HIV/Hep Therapy 2015. Abstract 63. 4. AASLD/IDSA. HCV guidance. September 2017. EBR/GZR[4]  ARVs without clinically significant interactions: 3TC, ABC, DTG, ENF, FTC, RAL, RPV, and TDF  Should NOT be used with COBI, EFV, ETR, NVP, or any HIV PI  Phase III study of EBR/GZR in HIV/HCV coinfection allowed 3TC, ABC, FTC, TDF, RAL, DTG, RPV[1]  EBR/GZR contraindicated or not recommended with all boosted ART regimens[2] – Marked increase in GZR exposure (potential for hepatotoxicity)  EFV results in a 80% reduction in GZR exposure, 50% decrease in EBR exposure[3]
  • 63. Case 3: Take-home Points  There has been an increase in injecting drug use in the United States and, with it, transmission of HCV  New direct-acting antiviral regimens for treating HCV infection are potent but differ in their potential for drug–drug interactions with ARV agents and other medications  Treatment of HIV/HCV coinfection may require consideration for the modification of HIV therapy during HCV treatment Slide credit: clinicaloptions.com
  • 64. Go Online for More CCO Coverage of HIV! Additional slidesets on contemporary management of HIV with expert faculty commentary Postconference clinical updates available following CROI, the International AIDS Conference, and IDWeek clinicaloptions.com/hiv

Editor's Notes

  1. Disclaimer: The materials published on the Clinical Care Options Web site reflect the views of the authors of the CCO material, not those of Clinical Care Options, LLC, the CME providers, or the companies providing educational grants. The materials may discuss uses and dosages for therapeutic products that have not been approved by the United States Food and Drug Administration. A qualified healthcare professional should be consulted before using any therapeutic product discussed. Readers should verify all information and data before treating patients or using any therapies described in these materials.
  2. ART, antiretroviral therapy; CVD, cardiovascular disease; HCV, hepatitis C virus. These slides include notes based on commentary provided by David A. Wohl, MD.
  3. CVD, cardiovascular disease.
  4. ART, antiretroviral therapy; CVD, cardiovascular disease. MJ is a 56-year-old African-American woman who was diagnosed with HIV 6 years prior but fell out of care about 1 year ago, and at that time she stopped her HIV therapy. A month ago she returned to care and was started on enalapril for hypertension. She smokes a pack of cigarettes per day and has done so for more than 30 years.
  5. ART, antiretroviral therapy; FTC, emtricitabine; TDF, tenofovir disoproxil fumarate. Her laboratory values at the time of her diagnosis 6 years ago included a CD4+ cell count that was almost 500 cells/mm3 and a viral load of 12,000 copies/mL. At that time she was started on a boosted PI plus fixed-dose FTC/TDF. Her viral load was suppressed, there were a few little blips here and there but nothing major, and then prior to falling out of care her CD4+ cell count had gone up to around 750 cells/mm3 and at that last viral load determination her viral load had been up, it had been suppressed but it was now 400 copies/mL. She stopped her HIV therapy after some personal tragedies; her father passed away and then she started caring for her 80-year-old mother, and that put a stress on her. Now her mom has moved to a nursing home and she has returned to care.
  6. BMI, body mass index; BP, blood pressure; HBV, hepatitis B virus; HCV, hepatitis C virus; HDL, high density lipoprotein cholesterol; LDL, low density lipoprotein cholesterol; MDRD, Modification of Diet in Renal Disease; TC, total cholesterol; TG, triglycerides. So summarizing a month ago and then today. A month ago her viral load, not surprisingly, was not suppressed at 21,000 copies/mL and her CD4+ cell count had dropped off with HIV therapy to 322 cells/mm3. Repeat testing for viral hepatitis revealed that her hepatitis C antibody was negative, her hepatitis B serology has proved that she was immune. Her serum creatinine was 1.13 mg/dL, which gives an MDRD calculated as greater than 60 mL/min. A viral load was done, as was mentioned, but also a genotype – given she was off of ART – and off of any selected pressure antiretrovirals her genotype resistance profile came back as wild-type.   At her current presentation, her blood pressure is elevated at 142/88 mm Hg and her BMI is 27. Urinalysis and other laboratories are listed here, some trace protein. Toxicology screen is negative. HLA-B*5701 is negative. Her lipid profile demonstrates that her HDL is a little low for a woman, total cholesterol is not bad, and LDL is 130 mg/dL.
  7. ART, antiretroviral therapy; FTC, emtricitabine; GT, genotype; HBV, hepatitis B virus; HCV, hepatitis C virus; HTN, hypertension; MDRD, Modification of Diet in Renal Disease; TDF, tenofovir disoproxil fumarate. So we will go over 3 main points, and the case is summarized on the bottom of this slide. Should she be prescribed a statin? And what we’re getting at here in this case is not only the management of someone who has challenges taking their HIV therapy and being engaged in care, but also at the same time is living with some very real comorbid risks, whether it be for cardiovascular disease or other things.   So there is a balance needed, we definitely can concentrate on her HIV therapy and management of her HIV infection, but we also have to pay some good attention, as well, to the comorbid conditions that may be just as threatening to her health. So one question is should she be prescribed a statin? Let us go over some of the information that would help us think about that.
  8. ART, antiretroviral therapy; CVD, cardiovascular disease; MI, myocardial infarction. There have been many cohort studies particularly looking at cardiovascular disease risk, and every conference has some updated data from these cohorts or new data from a new cohort. It is important that we consider that while there is lots of discussion about increased risk of cardiovascular disease among people living with HIV, which is true, people living with HIV clearly are at greater risk, it is complicated why they are at greater risk.   The trends over time are really interesting. If you look at the Kaiser Permanente cohort in California, we could see that over the last decade or so, MI rates and stroke rates have really gone down in HIV-infected participants in this managed care organization, as highlighted in blue, and really there is parity now between the HIV-positive and HIV-negative numbers of patients in this cohort. So while there has been an elevated risk, over time we can see a deceleration where the rates have gone down over time.   And the authors of this analysis posit that this is due to earlier ART initiation, more statin therapy to reduce cardiovascular risk, and also better management of hypertension and other comorbidities that contribute to cardiovascular disease. So I think this is really good news overall that shows that it is not that everyone living with HIV is a ticking time bomb and that cardiovascular disease is just inevitable, and we see that we can intervene and make a big difference.
  9. ASCVD, Atherosclerotic cardiovascular disease. In our case, one of the things that we would want to do is understand her risk for cardiovascular risk, and many clinicians are familiar with the Framingham Risk Score. The American College of Cardiology and the American Heart Association have collaborated to come up with a revised Risk Estimator. These tools are available online, available for downloading to your mobile device, and they provide a little bit different rubric for estimating risk and have been generally accepted to replace the Framingham calculator. And you’ll see that there are some differences.   The plug-ins are all entered here, so we put in her gender, her HDL cholesterol and her total cholesterol, which lets us calculate a non-HDL cholesterol. Of course, age. What is nice about this tool is that it also includes race, systolic blood pressure, that she is a smoker, and that she is being treated for hypertension.   What I like to do is say any guesses as to this patient’s risk?
  10. ASCVD, Atherosclerotic cardiovascular disease. So if we hit the button we obtain her calculated risk, and if you read the fine print, her 10-year risk for cardiovascular disease is at almost 20%, it’s astoundingly high. Her lifetime risk is 50%, but really in the blue is where we highlight. So she has a substantial risk of cardiovascular disease largely driven, of course, by her smoking.
  11. ASCVD, atherosclerotic cardiovascular disease; LDL, low density lipoprotein cholesterol. So if we read the recommendations that accompany the calculator, the idea is that adults who are 40 to 75 years of age, as our patient is, with an LDL of 70 to 189 mg/dL, with no diabetes, and with a 10-year risk of 7.5% or more should be treated with moderate- to high-intensity statin therapy, and this is a strong recommendation. So our patient clearly falls into the category of someone who should be offered a statin to reduce her cardiovascular risk based upon evidence.
  12. ACC/AHA, American College of Cardiology/American Heart Association; CVD, cardiovascular disease. What comes up oftentimes is how good are these estimators for people who are living with HIV. And there’s been a couple of different analyses of Framingham Risk and also the new ACC/AHA Cardiovascular Risk calculators, and in general what we see is that when we look at cohorts where we followed them over time and look at actual MI and stroke rates and look at the calculations that would have predicted these events, most of them underestimate the risk in cohorts of people living with HIV.   So when you see a risk of, for example, 10% or 8% in a person living with HIV, some experts feel like that is the minimum amount of risk that person has, and there may be some things that are not measured by the calculator that also will place that person at risk, whether it be the HIV itself or something else that is very hard to quantify as some unmeasured factor. And that is another point to consider when you are on the fence or in a gray area, it may tip you towards being a little bit more aggressive in your cardiovascular disease interventions, given that the calculators tend over different studies to underestimate actual risk.
  13. ART, antiretroviral therapy; FTC, emtricitabine; GT, genotype; HBV, hepatitis B virus; HCV, hepatitis C virus; HTN, hypertension; MDRD, Modification of Diet in Renal Disease; TDF, tenofovir disoproxil fumarate. So we have answered the question of if she should be on a statin; the answer is yes, we should at least try. What about her HIV therapy and how should we deal with that? She has been off of therapy for a while, she was on a boosted PI before, should we continue that or do something different?
  14. ABC, abacavir; ART, antiretroviral therapy; ATV, atazanavir; COBI, cobicistat; DRV, darunavir; DTG, dolutegravir; EFV, efavirenz; ETV, etravirine; EVG, elvitegravir; RAL, raltegravir; RPV, rilpivirine; RTV, ritonavir; TDF, tenofovir disoproxil fumarate; TAF, tenofovir alafenamide. This cartoon shows the spectrum wherein the boosted agents having more of a deleterious effect on lipids compared to drugs on the far left like TDF that may actually lower LDL cholesterol. We also have drugs in the middle that just looking strictly on the effects of lipids are much more neutral or benign compared to the boosters and even efavirenz.
  15. 3TC, lamivudine; ABC, abacavir; CV, cardiovascular; DTG, dolutegravir; ITT, intent to treat; RTV, ritonavir; VF, virologic failure.
  16. AE, adverse event; BL, baseline; CV, cardiovascular; d/c, discontinuation; DTG, dolutegravir; HDL-C, high density lipoprotein cholesterol; ITT, intent to treat; LDL-C, low density lipoprotein cholesterol; RTV, ritonavir; TC, total cholesterol; TG, triglycerides; VF, virologic failure.
  17. ART, antiretroviral therapy; ASCVD, Atherosclerotic cardiovascular disease; FTC, emtricitabine; GT, genotype; HBV, hepatitis B virus; HCV, hepatitis C virus; MDRD, Modification of Diet in Renal Disease; TDF, tenofovir disoproxil fumarate. Getting back to the case patient, what if instead of having a fairly normal renal function, MJ had a compromised renal function and her MDRD was calculated as 42 mL/min, how would that change the situation for those who said, well, I would put this person on TDF?
  18. ABC, abacavir; ART, antiretroviral therapy; ASCVD, Atherosclerotic cardiovascular disease; FTC, emtricitabine; GT, genotype; HBV, hepatitis B virus; HCV, hepatitis C virus; MDRD, Modification of Diet in Renal Disease; TDF, tenofovir disoproxil fumarate. One question that is important in cardiovascular disease and HIV is whether or not you would prescribe an abacavir-containing regimen. This may be attractive especially if we are not going to use TDF, but is abacavir a good option? And some people, of course, believe that abacavir has some issues as far as cardiovascular disease risk in contributing to that risk, and others are a little bit more unsure about that.
  19. ABC, abacavir; CVD, cardiovascular disease; DHHS, US Department of Health and Human Services; MI, myocardial infarction; RCTs, randomized controlled trials. To summarize a lot of data, there have been multiple studies that really have identified an increased risk of cardiovascular disease events with abacavir use. These have been mostly cohort studies, some analyses from randomized trials that were not designed specifically to look at cardiovascular disease endpoints. But there also have been studies that point the opposite way where they have not found evidence of an association, including meta-analyses of clinical trials, albeit those that do not follow people for many years, case-control studies, and cohort studies. So really there is a lot of confusion here.   In general, I think, given the potential for an association with abacavir, many providers avoid it in people with cardiovascular disease risk, whereas others are a little bit more comfortable using it and feel that any contribution might be minor or explained by biases or other confounders that could account for these findings within these studies. I think the other point, of course, is that now we are no longer limited to choosing between abacavir and tenofovir DF for patients with renal insufficiency and increased risk of cardiovascular disease. References: 1. Friis-Moller N, et al. Eur J Cardiovasc Prev Rehabil. 2010;17:491-501. 2. Friis-Moller N, et al. Eur J Prev Cardiol. 2016;23:214-223. 3. SMART/INSIGHT Study Group. AIDS. 2008;22:F17-F24. 4. Martin A, et al. Clin Infect Dis. 2009;49:1591-1601. 5. Durand M, et al. J Acquir Immune Defic Syndr. 2011;57:245-253. 6. Obel N, et al. HIV Med. 2010;11:130-136. 7. Choi AI, et al. AIDS. 2011;25:1289-1298. 8. Young J, et al. J Acquir Immune Defic Syndr. 2015;69:413-421. 9. Rotger M, et al. Clin Infect Dis. 2013;57:112-121. 10. Palella F, et al. CROI 2015. Abstract 749LB. 11. Lang S, et al. Arch Intern Med. 2010;170:1228-1238. 12. Ribaudo HJ, et al. Clin Infect Dis. 2011;52:929-940. 13. Bedimo RJ, et al. Clin Infect Dis. 2011;53:84-91. 14. Ding X, et al. J Acquir Immune Defic Syndr. 2012;61:441-447. 15. DHHS ART guidelines. October 2017.
  20. C, cobicistat; CKD-EPI, Chronic Kidney Disease Epidemiology Collaboration; F, emtricitabine; E, elvitegravir; eGFR, estimated glomerular filtration rate; TAF, tenofovir alafenamide; TDF, tenofovir disoproxil fumarate. And partly this is obviated by the development of tenofovir AF – which is a delivery system for the active drug of tenofovir, just like tenofovir DF is, but has much less impact on kidney function and bone mineral density. There have been a number of different studies now over the last 2-3 years that demonstrate not only the efficacy of tenofovir AF but also safety, including this study that shows that even people who are renally compromised can safely receive tenofovir AF. When we switch these patients to tenofovir AF, we do not observe a change in their GFR showing that there is not a deleterious effect even in patients whose renal function is already compromised.
  21. 3TC, lamivudine; ABC, abacavir; ART, antiretroviral therapy; ATV, atazanavir; CAB, cabotegravir; DRV, darunavir; DTG, dolutegravir; FTC, emtricitabine; IM, intramuscular; Ind, induction; LA, long acting; LPV, lopinavir; PO, orally; Q4W, every 4 weeks; Q8W, every 8 weeks; RAL, raltegravir; RPV, rilpivirine; RTV, ritonavir; TDF, tenofovir disoproxil fumarate; TAF, tenofovir alafenamide; VF, virologic failure. Others have looked at eliminating the NRTI drug class completely, and there has been a lot of work done over the years looking at tenofovir DF- and abacavir-sparing ART strategies, some using older drugs. And at the bottom of the table there are some novel combinations that we may hear more about, such as investigational cabotegravir and rilpivirine being studied as a long-acting injectable.   More recently there were data on dolutegravir and rilpivirine oral therapy, combining an INSTI and an NNRTI that showed high efficacy as a switch strategy for maintaining viral suppression. So you will see more of these types of studies where people are looking at avoiding the NRTI class completely. References: Cahn P, et al. EACS 2015. Abstract 961. Raffi F, et al. Lancet. 2014;384:1942-1951. Cahn P, et al. J Int AIDS Soc. 2017;20:1-7. Figueroa MI, et al. Abstract MOPEB0287. Taiwo BO, et al. Abstract MOAB0107LB. Joly V, et al. CROI 2017. Abstract 458. Perez-Molina JA, et al. Lancet Infect Dis. 2015;15:775-784. Di Giambenedetto S, et al. J Antimicrob Chemother. 2017;72:1163-1171. Arribas JR, et al. Lancet Infect Dis. 2015;15:785-792. Sued O, et al. IAS 2017. Abstract MOAB0106LB. Casado JL, et al. J Antimicrob Chemother. 2015;70:630-632. Margolis DA, et al. Lancet Infect Dis. 2015;15:1145-1155. Llibre JM, et al. CROI 2017. Abstract 44LB. Margolis DA, et al. Lancet. 2017;390:1499-1510.
  22. 3TC, lamivudine; ABC, abacavir; COBI, cobicistat; CrCl, creatinine clearance; DTG, dolutegravir; EFV, efavirenz; EVG, elvitegravir; FDA, US Food and Drug Administration; FTC, emtricitabine; RPV, rilpivirine; TAF, tenofovir alafenamide; TDF, tenofovir disoproxil fumarate. I think this is one of the most important slides of the deck and is really useful as a clip-and-save table, listing the single-tablet regimens on the left and the FDA-approved creatinine clearance cutoffs on the right. You can see that even as we go down to creatinine clearances of 30 mL/min, tenofovir AF is still FDA-approved for those patients. For the tenofovir DF-based non-cobicistat regimens and the fixed-dose dolutegravir/abacavir/lamivudine coformulation, the creatinine clearance cutoff is 50 mL/min. Most people starting therapy have higher values than these, but this is really good to know and refer to.
  23. ART, antiretroviral therapy; ATV, atazanavir; COBI, cobicistat; DRV, darunavir; DTG, dolutegravir; EFV, efavirenz; EVG, elvitegravir; FTC, emtricitabine; RAL, raltegravir; RPV, rilpivirine; RTV, ritonavir; TAF, tenofovir alafenamide; TDF, tenofovir disoproxil fumarate. The other issue, of course, is drug-drug interactions between ART and lipid-lowering therapy. In general, the way to think about this is that the boosters are going to boost the level of the statin, and then the inducers which include the NNRTIs like efavirenz and rilpivirine, are going to increase metabolism of the statin. So for any one statin, the effects can be 180-degrees different depending upon what ART is being administered at the same time. There are some statins we just cannot administer at all to people who are on anything that will boost the level, so lovastatin and simvastatin are contraindicated with any booster because the boosting is so profound that you get toxic levels of these statins when taken along with these HIV regimens that contain a pharmacological booster.   For others in the middle column, including atorvastatin, pravastatin, and rosuvastatin, they can be used and are commonly used, but you often have to titrate the dose. For example, with efavirenz, there will be induction and you might have to use a higher dose of these drugs. When you are using a booster, you have to be careful because there will be pharmacological boosting of these medicines, as well, so you use a lower drug dose. Pitavastatin generally does not need any adjustment at all, so you could use it as you would in someone without HIV infection, and then with rilpivirine there is really very little interaction fortunately with atorvastatin, so it can be dosed as you normally would with rilpivirine. So, again, another handy table that may be worth going over with your audience.
  24. ART, antiretroviral therapy; COBI, cobicistat; DTG, dolutegravir; RAL, raltegravir. Another important point is that statins are not equivalent to each other, and there are some statins – just like there are some HIV therapies – that are more potent. This is a dense table and I would not necessarily go over every detail, but some of the recommendations based upon cardiovascular risk include not only whether you should start a statin or not, but if you should start a high-intensity or a moderate-intensity statin. This table shows statins that are either more potent or can be dosed appropriately to be more potent. So potency, or intensity in this case, is based upon the chemical composition of the statin and also on the milligram amount that is used.   Atorvastatin or rosuvastatin are clearly standouts as being high-intensity. For moderate-intensity treatment, you can take low-intensity statins and dose them at a higher dose to get them to be basically moderate-intensity statins So, again, important when we look at some of the interpretations of our risk calculators to read the fine print, and it says not only should they be on a statin, but it may indicate – such as our case patient who is at quite a high risk – that they should be on a high-intensity or at least moderate-intensity statin. And this slide indicates how to dose this both with PIs that are boosted and also with NNRTI or raltegravir- or dolutegravir-containing regimens.
  25. ART, antiretroviral therapy; FTC, emtricitabine; GT, genotype; HBV, hepatitis B virus; HCV, hepatitis C virus; HTN, hypertension; MDRD, Modification of Diet in Renal Disease; TDF, tenofovir disoproxil fumarate. Moving on to the third question for discussion in this case, what else can be done to reduce MJ’s risk for new or worsening comorbidities? And, again, some salient features of her history are in the box as a reminder.  
  26. CT, computed tomography; DXA, dual-energy x-ray absorptiometry; STI, sexually transmitted infections. Here are some things to think about. Really important data show that the best things we can do for our patients are start them on HIV therapy for sure, but right on the heels of that is reducing their risk for serious comorbidities, such as cancer and cardiovascular disease. And we can probably do more by getting people to stop smoking and controlling their blood pressure and controlling their diabetes than probably anything else we could do for them as far as survival and avoidance of comorbidities.   Smoking cessation is essential. Is aspirin indicated for this patient? This is something that I think we do not really address as often as we should. The guidelines indicate that if you have a cardiovascular risk score of 10% or greater and do not have a contraindication to aspirin, low-dose aspirin is indicated. What about her blood pressure? I showed you before that she had a diastolic blood pressure of 88. Is that target?   What about chest CT, and this would be screening for lung cancer. That is recommended for people who have a 30-pack-year history and are over age 55, so she would qualify for low-dose radiation annual chest CTs to look for cancer. And, that is what a lot of primary care clinics and certainly a lot of pulmonary clinics are doing.   And avoiding future comorbidities by screening, so colonoscopy, Pap smear, and mammogram, of course. And there are guidelines that exist independent of HIV for this that are operative. DXA scanning, there is some guidance that we should be doing this more aggressively in people living with HIV, especially those who have had HIV long-term. Certainly perimenopausal women and maybe even men who are 50 years of age or older according to one expert panel.   Vaccinations, including the zoster vaccine, and then STI screening which is really important because the population you are treating may be enriched for those who are at higher risk for STIs. And so we should be thinking about that, and asking the right questions and screening appropriately so we can keep people healthy.
  27. ART, antiretroviral therapy; ARV, antiretroviral; ASCVD, Atherosclerotic cardiovascular disease; CVD, cardiovascular disease. The take-home points are we know that cardiovascular disease is more prevalent among people living with HIV. There are many potential reasons for that, which we do not cover in this presentation, but the full picture is not completely clear. I think it has a lot to do with the overlay of traditional risk factors – smoking, depression, stress, diet – these are all things that can contribute to cardiovascular disease. But we know that people with HIV have greater risk, so that is important.   The risk calculators are important, and if you cannot use the ASCVD, feel free to use the Framingham calculator. But ASCVD, I think, is being more widely adopted. And the threshold for statin initiation in general is about a 7.5% 10-year risk. Those with diabetes would pretty much qualify regardless of other factors. Different agents for HIV have varying effects on lipids, so it is important to become familiar with them and their drug interactions when we treat people, and I think HIV providers have done a good job of that. And then we always have to consider the impact of ARTs to some extent on cardiovascular risk, and we can fine-tune our selection of ART to make sure it does not get in the way or become a stumbling block to our efforts to reduce cardiovascular risk in our patients, which, again, is substantial.
  28. ART, antiretroviral therapy; BMD, bone mineral density.
  29. ART, antiretroviral therapy; BMD, bone mineral density; DRV, darunavir; FTC, emtricitabine; HBV, hepatitis B virus; HCV, hepatitis C virus; RTV, ritonavir; TDF, tenofovir disoproxil fumarate. The patient, FR, is a 62-year-old male living with HIV who has had a suppressed viral load for 5 years. His nadir CD4+ cell count was 105 cells/mm3, his baseline HIV genotype was wild-type. His initial ART was a boosted PI plus FTC/TDF, and he initiated ART during a clinical trial and has stayed on the same regimen ever since. He is HCV negative, HBV immune, and HLA-B*5701 negative.
  30. COPD, chronic obstructive pulmonary disease. His past medical history included COPD, no current medications. He used to smoke, but he does not any more. He does drink about 2-3 beers per night, he says.
  31. ASCVD, Atherosclerotic cardiovascular disease; DXA, dual-energy x-ray absorptiometry; MDRD, Modification of Diet in Renal Disease. At a routine visit 1 month ago, he came into the clinic and his arm was in a cast for a wrist fracture. He said that he had fallen on an icy walkway. His viral load is suppressed, his CD4+ cell count is pretty good at over 500 cells/mm3, his serum creatinine level is good at 0.9 mg/dL, and he has trace protein in his urine, which is fairly common. We calculated his 10-year ASCVD risk and now that he has stopped smoking, it is &amp;lt; 7.5%.   His vitamin D level is normal at 22 ng/mL. And then DXA scan was done and his L-spine T-score was a -2.4, his femoral neck was -2.6, and his hip was -2.5, and as we will discuss, those scores indicate that he has osteoporosis.
  32. ART, antiretroviral therapy; COPD, chronic obstructive pulmonary disease; DRV, darunavir; DXA, dual-energy x-ray absorptiometry; FTC, emtricitabine; GT, genotype; HBV, hepatitis B virus; HCV, hepatitis C virus; MDRD, Modification of Diet in Renal Disease; RTV, ritonavir; TDF, tenofovir disoproxil fumarate; WT, wild type. So what should we do? First question is what about his ART? As I mentioned, he is on a boosted PI and FTC/TDF; should that be modified? And then the second question for discussion is whether or not there is anything more that we should do for him, given the presentation?
  33. 3TC, lamivudine; ABC, abacavir; ART, antiretroviral therapy; ATV, atazanavir; BMD, bone mineral density; COPD, chronic obstructive pulmonary disease; Cr, creatinine; DRV, darunavir; DTG, dolutegravir; FTC, emtricitabine; GT, genotype; HBV, hepatitis B virus; HCV, hepatitis C virus; MDRD, Modification of Diet in Renal Disease; RPV, rilpivirine; RTV, ritonavir; TAF, tenofovir alafenamide; TDF, tenofovir disoproxil fumarate; WT, wild type. How would you manage his low BMD? A) continue his current ART therapy and start calcium and vitamin D with or without a bisphosphonate; B) switch FTC/TDF to ABC/3TC; C) switch FTC/TDF to FTC/TAF; D) switch DRV/RTV to boosted ATV or to an INSTI or RPV; D) switch to DTG plus RPV as a 2-drug NRTI-sparing regimen; or F) do something else.
  34. BMD, bone mineral density. We will go over some of the data, and I think in most cases, as we will point out with the next few slides, we would want to avoid continued TDF if we can avoid it, which I think we can, because TDF has been associated with lower bone density. And we will go over some of the differences between the other parts of his regimen and what their contribution might be to low bone density.   Taking a step back, just like we talked about cardiovascular disease, people with HIV do have higher risk for fractures. These are data published a while back that show that both men and women, especially as they get older, have a higher risk of fractures compared to people who are HIV-negative.  
  35. ART, antiretroviral therapy; BMD, bone mineral density. This is a nice illustration of what may be happening in people living with HIV compared to people without HIV, both men and women, where we get peak bone density in our early 20s, and then over time that starts to decrease very normally as a consequence of aging. But for men and women living with HIV, the declines are steeper in general. And, that may be due to the virus, it may be due to HIV therapy, it may be due to more corticosteroids or more alcohol or weight loss or low vitamin D. All sorts of things are mixed in to this and there is not one smoking gun that says this is all HIV or something else.   We do know that ART initiation in general, and particularly in people who start at lower CD4+ cell counts, is associated with decreases in bone density, which may be an immune reconstitution-related phenomenon. Does that happen as much in people who start at higher CD4+ cell counts? It seems not to, so this is likely a consequence of the timing of ART initiation that we have had over the previous 2 decades where people generally presented with CD4+ cell counts of 200-300 cells/mm3 and reconstituted from there. For those who are starting ART according to current guidelines with CD4+ cell counts much higher, in the 500-700 cells/mm3 range, I think there is less of an effect on bone density in most cases.
  36. ART, antiretroviral therapy; BL, baseline; BMD, bone mineral density; f/u, follow-up. The effect of ART on BMD is demonstrated nicely here with the START study. This is a study where people were randomized to start HIV therapy early vs later, and so these are all people with a CD4+ cell count &amp;gt; 500 cells/mm3. Patients who started on HIV therapy immediately in the blue had more of a decline in bone density than people who deferred their HIV therapy. Overall, waiting to start HIV therapy was not beneficial, there were more bad outcomes in people who waited to start therapy, but when you look at bone density, it was one outcome that was more favorable in those who waited. And, of course, a lot of this therapy was with TDF, which will decrease bone density.
  37. 3TC, lamivudine; ABC, abacavir; ATV, atazanavir; BMD, bone mineral density; DRV, darunavir; EFV, efavirenz; FTC, emtricitabine; RAL, raltegravir; RTV, ritonavir; TDF, tenofovir disoproxil fumarate. We know that different components of ART regimens can have differential effects on bone density, and this is illustrated nicely in this substudy of an ACTG trial in which patients were treated with either TDF/FTC or ABC/3TC, either in combination with efavirenz or atazanavir/ritonavir. A parsing out of the effects of each component is demonstrated in this 4x4 graphic in which spine BMD is shown in the top 2 graphs and hip BMD in the bottom 2 graphs. The results show that TDF certainly had more of an effect on decreasing bone density than ABC at both spine and hip. In addition, the boosted PI led to more of a decline in bone density in the spine vs efavirenz, but there was not as significant difference between the 2 components in the hip.
  38. ATV, atazanavir; BMD, bone mineral density; DRV, darunavir; FTC, emtricitabine; RAL, raltegravir; RTV, ritonavir; TDF, tenofovir disoproxil fumarate. Another ACTG study found a similar increased BMD loss with boosted PIs, but in this case comparing them to an INSTI, raltegravir. All patients in this study were receiving FTC/TDF as the NRTI pair. The results showed that boosted PIs were associated with increased bone density loss vs raltegravir. The results are somewhat complicated because TDF levels increase when it is given with a boosting agent. Therefore, patients in the PI arms were likely experiencing greater TDF exposure in addition to direct effects from the boosted PI itself, compared to patients in the raltegravir arm. So here we are probably seeing a combination of an enhanced TDF effect plus the boosted PI effect.
  39. AEs, adverse events; BMD, bone mineral density; C, cobicistat; E, elvitegravir; F, emtricitabine; TAF, tenofovir alafenamide; TDF, tenofovir disoproxil fumarate. Then we have TAF, and what is good about TAF is that when you look at bone and renal parameters, we see real differences from TDF. So, as I mentioned before, TAF is newer and it is another way of getting tenofovir into the cells. This slide shows mean spine and hip BMD changes for all patients in this large study, and also a subanalysis of younger patients who may be more susceptible to the bone effects of tenofovir. The results show that compared with TDF, TAF had much less of an effect on bone density, with overall minimal decreases. So TAF does appear to have a safer BMD profile vs TDF.
  40. ART, antiretroviral therapy; BL, baseline; BMD, bone mineral density; FTC, emtricitabine; TAF, tenofovir alafenamide; TDF, tenofovir disoproxil fumarate. When patients with virologic suppression are switched from TDF to TAF, BMD actually increases over time, which is quite remarkable.
  41. AEs, adverse events; ATV, atazanavir; BMD, bone mineral density; COBI, cobicistat; EVG, elvitegravir; FDA, US Food and Drug Administration; FTC, emtricitabine; RTV, ritonavir; TAF, tenofovir alafenamide; TDF, tenofovir disoproxil fumarate. A nice study done in virologically suppressed women looked at switching from a boosted PI plus FTC/TDF to the fixed-dose combination of EVG/COBI/FTC/TAF and showed favorable BMD changes with an increase in the hip and spine among those who switched compared with women who continued the boosted PI plus FTC/TDF regimen. For the spine, the difference between treatment arms was statistically significant.
  42. ART, antiretroviral therapy; BL, baseline; BMD, bone mineral density; DXA, dual-energy x-ray absorptiometry; ITT, intent to treat; TDF, tenofovir disoproxil fumarate; ZOL, zoledronic acid.
  43. ART, antiretroviral therapy; DTG, dolutegravir; FDA, US Food and Drug Administration; HBV, hepatitis B virus; ITT-E, intent to treat exposed; RPV, rilpivirine; VF, virologic failure.
  44. ART, antiretroviral therapy; COPD, chronic obstructive pulmonary disease; DRV, darunavir; DXA, dual-energy x-ray absorptiometry; FTC, emtricitabine; GT, genotype; HBV, hepatitis B virus; HCV, hepatitis C virus; RTV, ritonavir; TDF, tenofovir disoproxil fumarate; WT, wild type. The second part of discussion for this case is on any further interventions needed for this person.
  45. DXA, dual-energy x-ray absorptiometry; TDF, tenofovir disoproxil fumarate. Some recommendations indicate that we should be more aggressive about DXA scanning. There are issues of cost and accessibility, but DXA should be performed in all men 50 years of age or older, postmenopausal women, anyone who has a history of a fragility fracture, people who have received glucocorticoids, or people who are at high risk for falls. And then if bone density is found to be low, really not just ascribing this to HIV therapy or HIV, but really making sure that there are no secondary causes that could be modified such as profound vitamin D deficiency, hypothyroidism, hypogonadism.   We do give calcium and vitamin D; that alone can increase bone density. Bisphosphonates can also help along the way, and there have been ups and downs for bisphosphonates over time but better formulations are now available when needed. And we talked about avoidance of TDF, and I think that point has been well made.
  46. BMD, bone mineral density. The FRAX tool is a WHO-devised online calculator that incorporates multiple clinical risk factors to provide an estimate of fracture risk. Some data indicate that FRAX may underestimate fracture risk in people living with HIV and even in HIV-negative veterans in a large study.
  47. ART, antiretroviral therapy; BMD, bone mineral density; DXA, dual-energy x-ray absorptiometry. Low bone density is something we should be thinking about especially as people living with HIV get older. Traditional risk factors are probably the most operative, but there are HIV-related factors including ART. There are screening recommendations for bone density for men and women, especially for HIV-infected men and women; thinking about those and applying them, and then maybe being a little bit more aggressive if you can be. DXA scanning can be used, and in some instances the FRAX calculator can be adjunctive to that. And then always think about secondary causes of decreased bone density because many are treatable and may prevent you from taking other steps that may not be necessary to correct an underlying metabolic problem.
  48. GT, genotype; HCV, hepatitis C virus. Let’s move on the last case, a patient with HCV/HIV coinfection.
  49. GT, genotype; HCV, hepatitis C virus; IDU, injection drug use; WT, wild type. JA is a 46-year-old woman diagnosed with HIV and HCV coinfection within the last year. She is an active injection drug user, she does not take any comedications, she was screened for HIV after there was an outbreak among injection drug users in her community. Her HIV parameters at the time of diagnosis are listed on the left. She had a viral load of almost 60,000 copies/mL, and her CD4+ cell count was relatively high. Her HIV genotype results showed wild-type only virus. She has genotype 1b HCV coinfection with an HCV RNA level of 1.4 million IU/mL.
  50. ALT, alanine aminotransferase; AST, aspartate aminotransferase; HAV, hepatitis A virus; HBsAb, hepatitis B surface antigen antibody; Hct, hematocrit; MDRD, Modification of Diet in Renal Disease. Laboratory values are listed on this slide. They show good renal function, elevated transaminases, normal bilirubin, a low platelet count, and low albumin. She tests positive for hepatitis B surface antigen antibody and hepatitis A virus antibody, demonstrating HBV and HAV immunity. She has evidence of F3 fibrosis.
  51. ATV, atazanavir; COBI, cobicistat; FTC, emtricitabine; HCV, hepatitis C virus; TDF, tenofovir disoproxil fumarate. She has been receiving ATV/COBI plus FTC/TDF for the last 6 months. Her HIV-1 RNA level is suppressed and her CD4+ cell count has increased. She is still injecting but has been able to access clean needles through a local advocacy group. She has been taking her HIV therapy and she attends her clinic appointments. She was devastated by the diagnosis of HIV and HCV and tells you that she has heard a lot about HCV and would also like treatment for that infection.
  52. ART, antiretroviral therapy; ATV, atazanavir; COBI, cobicistat; FTC, emtricitabine; GT, genotype; HBV, hepatitis B virus; HCV, hepatitis C virus; MDRD, Modification of Diet in Renal Disease; TDF, tenofovir disoproxil fumarate; WT, wild type. In determining how best to manage her coinfections, we need to ask what hepatitis C therapy she should be prescribed and does her ART need to be modified, which is what we always have to think about when we are seeing patients with HCV/HIV coinfection.  
  53. ART, antiretroviral therapy; ESLD, end-stage liver disease; HBV, hepatitis B virus; HCV, hepatitis C virus. The NA-ACCORD looked at the risk for end-stage liver disease from 1996-2010. The results showed that in what was defined as the modern ART era, from 2006-2010, compared with HIV monoinfection, patients with HCV/HIV coinfection have a 5- to 6-fold increased risk. The same was true for patients with HBV/HIV coinfection, and those with HBV/HCV/HIV triple infection had an incidence ratio of nearly 17 for end-stage live disease risk; so this is a big problem for patients who are coinfected.  
  54. DCV, daclatasvir; GLE, glecaprevir; GT, genotype; LDV, ledipasvir; PIB, pibrentasvir; SOF, sofosbuvir; SVR12, sustained virologic response 12 weeks after treatment completion; VEL, velpatasvir. Fortunately, many studies show that HCV can be effectively treated and cured, including in people who have HIV coinfection. The results of HCV treatment studies in patients with HCV/HIV coinfection demonstrate equivalent efficacy rates relative to patients with HCV monoinfection. Therefore, although HIV coinfection is a risk factor for end-stage liver disease, it does not impede HCV cure rates in this group.
  55. 3TC, lamivudine; AASLD, American Association for the Study of Liver Diseases; ABC, abacavir; ATV, atazanavir; COBI, cobicistat; DCV, daclatasvir; DDI, drug–drug interaction; DRV, darunavir; DSV, dasabuvir; DTG, dolutegravir; EBR, elbasvir; EFV, efavirenz; EVG, elvitegravir; ETR, etravirine; GLE, glecaprevir; GZR, grazoprevir; LDV, ledipasvir; PIB, pibrentasvir; PTV, paritaprevir; RAL, raltegravir; RPV, rilpivirine; RTV, ritonavir; SMV, simeprevir; SOF, sofosbuvir; TAF, tenofovir alafenamide; TDF, tenofovir disoproxil fumarate; VEL, velpatasvir. The challenge for clinicians is that there are multiple important drug-drug interactions between antiretroviral drugs and direct-acting antiviral drugs for HCV therapy, requiring careful selection of either an HCV therapy that will work safely with whatever HIV therapy the patient is already receiving or of a modified ART regimen that will be compatible with the HCV therapy we need to use, or both.   There is a lot of green on this table, indicating combinations that are not anticipated to have a clinically significant interaction, but there is also a lot of orange, indicating that the intersecting regimens/drugs should not be coadministered. The boosting agents, ritonavir and cobicistat, have larger regions of orange than many other antiretroviral agents because they tend to have more frequent drug-drug interactions as a result of their pharmacologic boosting properties.
  56. HCV, hepatitis C virus. So let’s go back to our case patient who has genotype 1b HCV/HIV coinfection, and we find out that her insurance will only cover elbasvir/grazoprevir for 12 weeks of therapy for her HCV infection, and that if you want to use anything else you will need to file for an appeal, which may or may not be successful.
  57. 3TC, lamivudine; ABC, abacavir; ART, antiretroviral therapy; ATV, atazanavir; COBI, cobicistat; DDIs, drug-drug interactions; DTG, dolutegravir; DRV, darunavir; EBR, elbasvir; EVG, elvitegravir; FTC, emtricitabine; GZR, grazoprevir; GT, genotype; HBV, hepatitis B virus; HCV, hepatitis C virus; MDRD, Modification of Diet in Renal Disease; QD, once daily; RAL, raltegravir; TAF, tenofovir alafenamide; TDF, tenofovir disoproxil fumarate; WT, wild type. Given that information, our question is, how would you manage her ART – atazanavir/cobicistat plus FTC/TDF – to avoid potential drug-drug interactions when treating her hepatitis C with EBR/GZR? The answer options are: A) stay on the current regimen; B) switch to DRV plus RTV plus FTC/TAF; C) switch to EVG/COBI/FTC/TAF; D) switch to DTG/ABC/3TC; E) switch to RAL QD plus FTC/TAF; or F) something else.   The correct answer is D or E. We want to avoid any of the boosters and any other drugs with the potential to interact with her hepatitis C therapy and lead to toxicity.  
  58. 3TC, lamivudine; ABC, abacavir; ART, antiretroviral therapy; COBI, cobicistat; DTG, dolutegravir; EBR, elbasvir; EFV, efavirenz; ENF, enfuvirtide; ETR, etravirine; FTC, emtricitabine; GZR, grazoprevir; NVP, nevirapine; RAL, raltegravir; RPV, rilpivirine; TDF, tenofovir disoproxil fumarate. For recommendations regarding the use of EBR/GZR in combination with ART, there have been a number of studies that have shown that there are some safer ART options, and they are listed on this slide. Boosting agents should not be used, but unboosted INSTIs are safe to use, as is rilpivirine, abacavir, lamivudine, emtricitabine, and tenofovir AF or DF. Efavirenz leads to an 80% reduction in GZR exposure and a 50% reduction in EBR exposure, so it should not be used in combination with this HCV regimen.   The box at the bottom of this slide summarizes the antiretroviral agents without clinically significant interactions with EBR/GZR and those that should not be used in combination with EBR/GZR.
  59. We know that there has been an increase in opioid use and injection drug use in general, and with it there has been some resurgence of hepatitis C especially in certain geographic locations. The new DAAs are fantastic for treating hepatitis C, but there are important drug-drug interactions with antiretroviral drugs that we really have to be very mindful of. The treatment of HCV/HIV coinfection may require modification of HIV therapy, and sometimes this is a good way to update your HIV therapy and modernize or simplify it. In other cases it may be temporary and you are going to have to switch back. In general, though, I think cotreatment of these 2 infections can be accommodated well.